Exam 2 (Ch 7, 13-19, 29, 33, 37-39) Practice Questions Flashcards

1
Q

A nursing professor pulls a student aside to discuss documenting a patient’s blood pressure of 202/122 but not reporting this to the primary nurse. When discovered, the patient was transferred to the intensive care unit for treatment and monitoring. How does the faculty best explain to the student that their inaction reflects negligence?

A. “You did not re-assess your patient.”
B. “There was poor interprofessional communication with the health care team.”
C. You failed to act as a reasonably prudent nurse would under similar circumstances.”
D. “This action is consistent with a felony criminal action.”

A

C.

RATIONALE:
c. Negligence is defined as performing an action that the reasonably prudent nurse would not perform or failing to act as a reasonably prudent nurse would in similar circumstances. Negligence may be an act of omission or commission. Criminal law concerning state and federal criminal statutes includes murder, manslaughter, criminal negligence, theft, and illegal possession of drugs. Public law regulates relationships between people and the government. Private or civil law includes laws relating to contracts, ownership of property, and the practice of nursing, medicine, pharmacy, and dentistry.

How well did you know this?
1
Not at all
2
3
4
5
Perfectly
2
Q

Nursing students approaching graduation and licensure are required to read the state nurse practice act. Which topics in the law will they identity as guides to professional practice? Select all that apply.

A. Actions resulting in discipline
B. Clinical procedures
C. Medication administration
D. Scope of practice
E. Delegation policies
F. Medicare reimbursement

A

A,D

RATIONALE:
a, d. Each state has a nurse practice act that protects the public by broadly defining the legal scope of nursing practice. Practicing beyond those limits makes nurses vulnerable to charges of violating the state nurse practice act. Nurse practice acts also list the violations that can result in disciplinary actions against nurses. Clinical procedures are covered by the health care institutions themselves. Medication administration and delegation are topics covered by the board of nursing. Laws governing Medicare reimbursement are enacted through federal legislation.

How well did you know this?
1
Not at all
2
3
4
5
Perfectly
3
Q

A nurse on a surgical unit is concerned about a colleague’s possible substance use disorder. Which signs and symptoms could support the nurse’s suspicion? Select all that apply.

A. Exhibiting diminished alertness and somnolence while working
B. Attending multiple continuing education conferences
C. Offering to medicate coworkers’ patients for pain
D. Making incorrect narcotics counts and creating wastage
E. Leaving the unit frequently

A

A, C, D, E

RATIONALE:
a, c, d, e. Signs of substance use in nurses may include diminished alertness or somnolence, leaving the unit frequently, incorrect narcotic counts, wastage, offers to medicate colleagues’ patients, or changes in job performance, among others. Attending professional conferences is an example of a nurse who is fully engaged with their work.

How well did you know this?
1
Not at all
2
3
4
5
Perfectly
4
Q

A new graduate nurse tells the preceptor they want to obtain recognition in wound care, a specialty area of nursing. What credential will this nurse need to seek?

A. Accreditation
B. Licensure
C. Certification
D. Board approval

A

C

RATIONALE:
c. Certification is the process by which a person who has met certain criteria established by a nongovernmental association is granted recognition in a specified practice area. Nursing is one of the groups operating under state laws that promote the general welfare by determining minimum standards of education through accreditation of schools of nursing. Licensure is a legal document that permits a person to offer to the public skills and knowledge in a particular jurisdiction, where such practice would otherwise be unlawful without a license. State board of approval ensures that nurses have received the proper training to practice nursing.

How well did you know this?
1
Not at all
2
3
4
5
Perfectly
5
Q

The nurse reports to their manager that informed consent was not obtained from a patient for whom HIV testing was already performed. The nurse suggests which intentional tort may have been committed?

A. Assault
B. Battery
C. Invasion of privacy
D. False imprisonment

A

B

RATIONALE:
b. Assault is a threat or an attempt to make bodily contact with another person without that person’s consent. Battery is an assault that is carried out. Every person is granted freedom from bodily contact by another person unless consent is granted. The Fourth Amendment gives citizens the right of privacy and the right to be left alone; a nurse who disregards these rights is guilty of invasion of privacy. Unjustified retention or prevention of the movement of another person without proper consent can constitute false imprisonment.

How well did you know this?
1
Not at all
2
3
4
5
Perfectly
6
Q

A patient died during routine outpatient surgery, and the nurse was accused of having failed to monitor and interpret vital signs. Which fact must be established to prove them guilty of malpractice or negligence?

A. The surgeon testifies the nurse’s action was pure negligence, saying that the patient could have been saved.
B. This patient should not have died since they were healthy, physically active, and involved in the community.
C. The nurse intended to harm the patient and was willfully negligent, as evidenced by the tragic outcome.
D. The nurse had a duty to monitor the patient, and due to the nurse’s failure to perform this duty, the patient died.

A

D

RATIONALE:
d. Liability involves four elements that must be established to prove that malpractice or negligence has occurred: duty, breach of duty, causation, and damages. Duty refers to an obligation to use due care (what a reasonably prudent nurse would do) and is defined by the standard of care appropriate for the nurse–patient relationship. Breach of duty is the failure to meet the standard of care. Causation, the most difficult element of liability to prove, shows that the failure to meet the standard of care (breach) caused the injury. Damages are the actual harm or injury resulting to the patient.

How well did you know this?
1
Not at all
2
3
4
5
Perfectly
7
Q

An attorney representing a patient’s family who is suing for wrongful death calls the nurse to obtain a better understanding of the nurse’s actions. How will the nurse respond?

A. “I can’t talk with you; you will have to contact my attorney.”
B. “I will answer your questions, so you’ll understand how the situation occurred.
C. “I hope I won’t be blamed for the death because it was so busy that day.”
D. “First tell me why you are doing this to me. This could ruin my career!”

A

A

RATIONALE:
a. The nurse should not discuss the case with anyone at the facility (except the risk manager), with the plaintiff, with the plaintiff’s lawyer, with anyone testifying for the plaintiff, or with reporters. This is one of the cardinal rules for nurse defendants.

How well did you know this?
1
Not at all
2
3
4
5
Perfectly
8
Q

A nurse follows a prescription written by the health care provider to administer a medication to which the patient is allergic. How does the nurse interpret their liability for administering this medication?

A. The nurse is not responsible because they were following the provider’s orders.
B. The nurse is responsible because they administered the medication.
C. The health care provider is responsible because they ordered the drug.
D. The nurse, health care provider, and pharmacist bear responsibility for their actions.

A

D

RATIONALE:
d. Nurses are legally responsible for carrying out the orders of the health care provider in charge of a patient unless an order would lead a reasonable person to anticipate injury if it was carried out. If the nurse should have anticipated injury and did not, both the prescribing health care provider and the administering nurse are responsible for the harms to which they contributed.

How well did you know this?
1
Not at all
2
3
4
5
Perfectly
9
Q

A nurse answers a call light and finds the patient on the floor. After the health care provider examines the patient and finds no injury, the nurse returns the patient to bed and fills out an incident report. What statements are true about incident reports? Select all that apply.

A. They can be used as disciplinary action against staff members.
B. They can be used as a means of identifying risks.
C. They can be used for quality control.
D. They must be completed by the facility manager.
E. They make facts available in litigation cases.
F. They should be documented in the patient record.

A

B,C,E

RATIONALE:
b, c, e. Incident reports are used for quality improvement and should not be used for disciplinary action against staff members. They are a means of identifying risks and are filled out by the nurse responsible for the injured party. An incident report makes facts available in case litigation occurs; in some states, incident reports may be used in court as evidence. A health care provider completes the incident form with documentation of the medical examination of the patient, employee, or visitor with an actual or potential injury. Documentation in the patient record should not include the fact that an incident report was filed.

How well did you know this?
1
Not at all
2
3
4
5
Perfectly
10
Q

A nursing student is preparing to administer medications and asks the clinical instructor about legal liability in clinical practice. What is the most appropriate response?

A. “Students are not responsible for their acts of negligence resulting in patient injury.”
B. “Student nurses are held to the same standard of care that would be used to evaluate the actions of a registered nurse.”
C. “Hospitals are exempt from liability for student negligence if the student nurse is properly supervised by an instructor.”
D. “Most nursing programs carry group professional liability making student personal professional liability insurance unnecessary.”

A

B

RATIONALE:
b. Student nurses are held to the same standard of care that would be used to evaluate the actions of a registered nurse. Student nurses are responsible for their own acts of negligence if these result in patient injury. A hospital may also be held liable for the negligence of a student nurse enrolled in a hospital-controlled program because the student is considered an employee of the hospital. Nursing instructors may share responsibility for damages in the event of patient injury if an assignment called for clinical skills beyond a student’s competency or the instructor failed to provide reasonable and prudent clinical supervision. Most nursing programs require students to carry personal professional liability insurance.

How well did you know this?
1
Not at all
2
3
4
5
Perfectly
11
Q

The nurse manager reviews the medical record of a patient who has accused a nurse of negligence after requiring a “needless” admission to the intensive care unit postoperatively. Which entry in the electronic health record requires follow-up by the manager?

Exhibit: Electronic health record (EHR)
Nursing Notes: Postoperative follow-up
12:20 pm: patient still reporting incisional pain of 10/10, provider contacted, increased morphine from 1 mg to 2 mg every hour
2: 15 pm: dime-sized, dark red–brown blood stain on dressing; area circled
2:30 pm: patient reports incisional pain, 7/10, 2 mg morphine administered
2:45 pm: vital signs T 99.2°, P 120, RR 20, BP 84/48; will continue to monitor

A. Inappropriately recorded vital signs
B. Pain treated without appropriate assessment
C. Failure to follow up on tachycardia and hypotension
D. Lack of interpretation of vital signs and follow-up

A

D

RATIONALE:
d. Nurses are responsible for gathering assessment data including vital signs and interpreting them considering the patient’s condition and trends. The nurse did not document interventions from the health care provider for typical symptoms of shock, including tachycardia and hypotension.

How well did you know this?
1
Not at all
2
3
4
5
Perfectly
12
Q

A patient admitted through the emergency department for a severe infection is receiving intravenous (IV) antibiotics. The patient, who has been oriented, demands the nurse remove the IV because the patient is leaving now. What action will the nurse take?

A. Apply soft wrist restraints
B. Perform a neurologic assessment
C. Explain that after signing an “against medical order form,” the patient may leave
D. Call the patient’s family to encourage the patient to stay

A

C

RATIONALE:
c. A person cannot be legally forced to remain in a health facility, such as a hospital, if that person is of sound mind. The patient signs an “against medical orders” form when insisting on being discharged, to indicate not holding the facility responsible for harm from leaving. Applying soft wrist restraints when the patient has expressed wanting to leave constitutes battery, which includes willful, angry, and violent or negligent touching of another person’s body or clothes or anything attached to or held by that other person. The patient has been oriented, so another assessment is not indicated. The patient, not the family, has autonomy.

How well did you know this?
1
Not at all
2
3
4
5
Perfectly
13
Q

A nursing student is committed to providing thoughtful, person-centered care. Which nursing actions demonstrate this type of care? Select all that apply.

A. Assisting patients to select meals based on their cultural observances
B. Providing nursing care based on patients’ needs and preferences
C. Documenting nursing interventions in the electronic health record
D. Reviewing fingerstick blood glucose levels with the primary nurse
E. Listening to a patient’s concern for their ill significant other

A

A, B, E

RATIONALE:
a, b, e. The nursing process ensures that nurses are person centered rather than task centered. Attending to cultural preferences and needs and listening to a patient’s concerns are patient-centered actions. Documentation and communication with other members of the health care team are not specifically patient centered.

How well did you know this?
1
Not at all
2
3
4
5
Perfectly
14
Q

A patient who is receiving cancer chemotherapy tells the nurse, “The treatment for this cancer is worse than the disease itself. I’m stopping treatment.” Which nursing action best promotes a patient-centered, therapeutic relationship?

A. Determining if the patient database is adequate to address the problem
B. Considering whether to suggest a counseling session for the patient
C. Reassessing the patient and determining how to best support them
D. Identifying possible interventions and critiquing the merit of each option

A

C

RATIONALE:
c. Reassessing the patient allows the nurse and patient to clarify the patient’s goal(s) and develop interventions to best meet them. Once the problem is addressed, it is important for the nurse to judge the adequacy of the knowledge, identify potential problems, use helpful resources, and critique the decision.

How well did you know this?
1
Not at all
2
3
4
5
Perfectly
15
Q

The nursing philosophy in an acute care hospital includes a commitment to deliver thoughtful, person-centered care. Which description of the nursing process best supports this commitment?

A. Systematic
B. Interpersonal
C. Dynamic
D. Universally applicable in nursing situations

A

B

RATIONALE:
b. Interpersonal. All other options are characteristics of the nursing process but focus on the patient best illustrates the interpersonal dimension of the nursing process.

How well did you know this?
1
Not at all
2
3
4
5
Perfectly
16
Q

A staff nurse tells a new graduate nurse not to bother studying too hard, since most clinical reasoning becomes second nature and intuitive once they begin practicing. Which response by the student is appropriate?

A. Intuitive problem solving comes with years of practice and observation based on nursing knowledge and science.
B. For nursing to remain a science, nurses must continue to be vigilant about avoiding intuitive reasoning.
C. The emphasis on logical, scientific, evidence-based reasoning has held nursing back; we need intuitive, creative thinkers.
D. The nurse’s preference dictates whether they are logical, scientific thinkers or intuitive, creative thinkers.

A

A

RATIONALE:
a. When intuition is used alone, increased risks and fewer benefits may occur. Beginning nurses must use nursing knowledge and scientific problem solving as the basis of care; intuitive problem solving comes with years of practice and observation. If the beginning nurse has an intuition about a patient, that information should be discussed with the faculty member, preceptor, or supervisor. There is a place for intuitive reasoning in nursing, but it will augment, not replace logical, scientific reasoning. Critical thinking is contextual and changes depending on the circumstances, not on personal preference.

How well did you know this?
1
Not at all
2
3
4
5
Perfectly
17
Q

The nurse uses blended competencies when caring for patients in a rehabilitation facility. Which interventions reflect the use of cognitive skills? Select all that apply.

A. Monitoring for side effects of medications
B. Safely administering an injectable medication
C. Teaching a patient about diabetes and its management
D. Acting as witness by signing a surgical consent form
E. Helping a patient identify their progress in physical therapy
F. Comforting a patient who has received bad news

A

A,C

RATIONALE:
a, c. Using critical thinking to teach a patient about a disease process and management and monitoring for side effects of medications are cognitive competencies. Performing an injection correctly is a technical skill; witnessing/signing an informed consent form is a legal/ethical action, and comforting a patient is an interpersonal skill.

How well did you know this?
1
Not at all
2
3
4
5
Perfectly
18
Q

A nurse uses critical-thinking skills to develop the care plan for an older adult with dementia awaiting placement in a long-term care facility. Which statements describe characteristics of the critical thinking used by nurses engaged in clinical reasoning? Select all that apply.

A. Functions independently of nursing standards, ethics, and state practice acts
B. Based on the principles of the nursing process, problem solving, and the scientific method
C. Driven by patient, family, and community needs as well as nurses’ needs to give competent, efficient care
D. Avoids designs to compensate for problems created by human nature, such as medication errors
E. Constantly reevaluating, self-correcting, and striving for improvement
F. Focuses on the big picture rather than identifying the key problems, issues, and risks involved with patient care

A

B,C,E

RATIONALE:
b, c, e. Critical thinking applied to clinical reasoning and clinical judgment is guided by standards, policies and procedures, and ethics. When applying principles of nursing process, problem solving, and the scientific method, clinical reasoning identifies the key problems, issues, and risks. This is driven by patient, family, and community needs as well as nurses’ needs to give competent, efficient care. It also calls for strategies that make the most of human potential and compensate for problems created by human nature. It is constantly reevaluating, self-correcting, and striving to improve the quality and safety of health care systems (Alfaro-LeFevre, 2014).

How well did you know this?
1
Not at all
2
3
4
5
Perfectly
19
Q

A nurse is caring for a patient with type 2 diabetes who has an infected foot ulcer requiring dressing changes. Which nursing action best demonstrates the QSEN competency of patient-centered care?

A. Asking the patient if they would like their spouse to be present for a teaching session
B. Researching new procedures to care for foot ulcers when developing a care plan for this patient
C. Leading a grand rounds or unit-based discussion on complications of diabetes
D. Using the electronic medical record to review trends of the patient’s blood glucose levels

A

A

RATIONALE:
a. Patient-centered care commits to developing caring relationships based on mutual trust to communicate and deliver care based on patient preferences and values. Evidence-based practice integrates the best current evidence for safe practice with clinical expertise. Teamwork and collaboration shares patient information or opportunities for learning with others. Informatics manages patient information, mitigates error, and supports decision making using the electronic medical record and other databases.

How well did you know this?
1
Not at all
2
3
4
5
Perfectly
20
Q

The nursing assessment of a patient with a diagnosis of anorexia nervosa reveals the patient consumes a vegan diet of 700 calories daily and has lost 30 lb in 4 months. The nurse’s recommendation to meet with a nutritionist is the outcome of which process?

A. Clinical judgment
B. Nursing process
C. Clinical reasoning
D. Critical thinking

A

A

RATIONALE:
a. Clinical judgment is the outcome of critical thinking and clinical reasoning, using the nursing process as a framework. Clinical reasoning refers to ways of thinking about patient care issues including weighing and validating options (determining, preventing, and managing patient problems). Critical thinking includes reasoning both outside and inside of the clinical setting.

How well did you know this?
1
Not at all
2
3
4
5
Perfectly
21
Q

A nurse working in a long-term care facility reviews the electronic health records of patients who have fallen in the last month to determine if there is a common risk factor. Which QSEN competency is the nurse demonstrating?

A. Patient-centered care
B. Evidence-based practice
C. Teamwork and collaboration
D. Informatics

A

D

RATIONALE:
d. Informatics uses information and technology to communicate, manage knowledge, mitigate error, and support decision making. Thoughtful, patient-centered care emphasizes recognition of the patient or designee as the source of control and full partner in compassionate and coordinated care, based on respect for patients’ preferences, values, and needs. Evidence-based practice integrates the best current evidence with clinical expertise and patient and family preferences and values to deliver optimal health care. Teamwork and collaboration refer to effective functioning within nursing and interprofessional teams, fostering open communication, mutual respect, and shared decision making to achieve quality patient care.

How well did you know this?
1
Not at all
2
3
4
5
Perfectly
22
Q

A new graduate nurse phones the surgeon to report their patient is having severe incisional pain. The surgeon asks about vital signs and appearance of the wound, causing the nurse to return to the bedside for additional assessments. Upon reflection with the preceptor, which characteristic of the nursing process should the nurse have remembered?

A. Centric
B. Dynamic
C. Interpersonal
D. Systematic

A

D

RATIONALE:
d. The nursing process is systematic, iterative, and overlapping. By reporting an isolated symptom, the nurse has overlooked the benefit of systematic and inclusive assessment. While the nursing process is presented as an orderly progression of phases, there is a dynamic interaction and flow of phases into one another.

How well did you know this?
1
Not at all
2
3
4
5
Perfectly
23
Q

The nurse is formulating a care plan for a patient in a long-term care facility who has lost 12 lb in the last 2 months. To arrive at a patient-centered nursing judgment, what will the nurse do first?

A. Ensure the patient is receiving foods they like, including favorites.
B. Make sure the patient’s dentures are clean and inserted at mealtimes.
C. Assess the patient’s food intake and hydration over the last 1 to 3 days.
D. Request that the nursing assistant feed the client at mealtime.

A

C

RATIONALE:
c. The nurse uses the nursing process to arrive at a clinical judgment. After analyzing the assessment data, the nurse determines, through clinical reasoning, whether the related factors in the patient’s weight loss, such as dislike of menu options, lack of dentition, or inability to perform activities of daily living such as feeding, should be the focus of interventions.

How well did you know this?
1
Not at all
2
3
4
5
Perfectly
24
Q

When implementing a thoughtful, patient-centered care plan, which action does the nurse prioritize?

A. The patient’s loved ones are considered part of the team.
B. A caring relationship with mutual trust is established.
C. Measures for safety are visibly incorporated.
D. Transparent communication is observed.

A

C

RATIONALE:
c. Although developing a thoughtful, patient-centered approach is focused on caring and mutual trust, the nurse uses the nursing process and Maslow’s hierarchy of needs to prioritize care. Safety is a higher-level need than love and belonging, and therefore the priority.

How well did you know this?
1
Not at all
2
3
4
5
Perfectly
25
Q

An oncology nurse is analyzing a patient’s strengths and finds the patient is well educated, learns quickly, and is resilient. In which phase of the nursing process will the nurse use this information?

A. Diagnosing
B. Evaluating
C. Planning
D. Implementing

A

A

RATIONALE:
a. Assessing for strengths and weaknesses is the first step of the nursing process, which has been completed. Next, the nurse clusters cues and develops diagnoses that give rise to interventions. Evaluating the plan is followed by completing or modifying the plan.

How well did you know this?
1
Not at all
2
3
4
5
Perfectly
26
Q

A student nurse walks into a patient room, introduces themselves, and begins to complete a full head-to-toe assessment. The clinical faculty member enters the room, introduces themselves, and asks the student to step out of the room for a moment. The student meets the faculty member in the hallway and is asked to identify 15 cues or observations they noted during their initial contact with the patient and the patient’s environment. Although the student is unable to reach 15 observations, the faculty guides the student to recognize the linen on the floor, old dinner tray on the windowsill, empty water pitcher, twisted oxygen tubing, the patient’s pallor, and several other things requiring action. What is the value of engaging in this kind of activity with students in the clinical setting?

A. Developing situational awareness is important to risk prevention, timely implementation of interventions, and prioritizing actions
B. Managing cognitive load begins with systematically sorting mental images and immediately addressing pressing concerns
C. Nursing best practice requires that an environmental scan be completed and documented in the electronic health record (EHR)
D. Designing interventions that increase patient satisfaction is an essential focus for nurses when completing their initial assessments

A

A

RATIONALE:
a. The case represents a concrete example of how students develop situational awareness and how that awareness develops with experience. Cognitive load management occurs in many ways; the use of mental images may not facilitate cognitive load management. Although an environmental scan is important, it is generally not documented in the EHR. Patient satisfaction is not the priority when completing the initial assessment; rapport formation, solid assessments, and good patient outcomes are associated with patient satisfaction.

How well did you know this?
1
Not at all
2
3
4
5
Perfectly
27
Q

A nursing program uses Tanner’s Clinical Judgment Model, a research-based model that accounts for differences in the patient, environment, and individual student nurse. What makes Tanner’s reflection step unique?

A. The emphasis is on noticing, interpreting, and responding; reflection is less important.
B. Reflection occurs both in-action (in the moment) and on-action (after the situation).
C. Reflection occurs first in the model that is focused on rapid decision making and patient outcomes.
D. Reflection is the last step in a linear model and is designed to minimize bias in the student nurse.

A

B

RATIONALE:
b. Reflection drives the clinical judgment cycle and allows for the integration of new knowledge that will inform future situations. Reflection is as important in Tanner’s model as the other elements of noticing, interpreting, and responding. The focus of the model is the development of clinical judgment, rather than rapid decision making; as clinical judgment is developed, decision making improves and may lead to better patient outcomes. Tanner’s model is cyclic, not linear; although bias may be addressed as part of the development of clinical judgment, it is not the focus.

How well did you know this?
1
Not at all
2
3
4
5
Perfectly
28
Q

Nursing programs prepare students for safe clinical practice. As a student nurse, why is a basic understanding of NCSBN’s Clinical Judgment Measurement Model (CJMM) important? Select all that apply.

A. Successful completion of the NCLEX is required for professional licensure in the United States.
B. Nurse educators use the CJMM model and NCLEX test plans to develop exam questions.
C. Students should be intimately familiar with theoretical models of education to answer questions.
D. Appreciation of the core principles assists students in understanding the structure and intent of nursing exams.
E. There is overlap in the core components of clinical judgment models, measurement models, and the nursing process.

A

A,B,D,E

RATIONALE:
a, b, d, e. Nurses in the United States must pass NCLEX prior to being issued a license to practice as a professional nurse. To help students achieve this goal, nurse educators model course exams on the NCSBN’s NCLEX test plan in terms of content and style. Understanding the why (rationale for actions) is often helpful when students are engaged in studying, working to apply the information they have learned, and developing test-taking strategies. There is overlap between the models and processes identified as foundational to nursing education, which demonstrates the importance of fundamental concepts. A deep understanding of theoretical and philosophical models is not necessary for student nurses.

How well did you know this?
1
Not at all
2
3
4
5
Perfectly
29
Q

Nursing students and those studying other health sciences (medicine, pharmacy, physical therapy, etc.) are often engaged in competency-based education. What is the value of competency-based education?

A. It provides comprehensive skills checklists for students to check their progress and move on to other elements.
B. It allows for student individualization based on their unique experience and preferences.
C. It provides specific guidance on the expected level of performance that integrates knowledge, skills, abilities, and judgment.
D. Like most other education models, it is a high-level way of thinking that is not related to clinical judgment.

A

C

RATIONALE:
c. The definition of a competency included in the Nursing: Scope and Standards of Practice (ANA, 2021) identified includes knowledge, skills, abilities, and judgment. Competency-based education is more than a checklist and often requires repeated exposure to concepts for mastery in a variety of contexts. Although the delivery of education can flex to meet the needs of students, testing is standardized to address core competencies. Competency-based education is very direct and concrete; the development of competence requires clinical judgment.

How well did you know this?
1
Not at all
2
3
4
5
Perfectly
30
Q

The development of clinical judgment requires intentional focus and a willingness to grow and change both personally and professionally. How can a nursing student best foster the development of clinical judgment?

A. Engaging in learning that only appeals to their preferred learning style
B. Focusing on knowledge acquisition that is straightforward and clear
C. Developing a model for learning that integrates feedback and reflection
D. Focusing inward to develop emotional intelligence and communication skills

A

C

RATIONALE:
c. Each nursing student is in charge of their learning, including integrating feedback from exams, clinical experiences, simulations, and other assignments—reflecting on what has been learned and integrating new learning. Adaptability is key. Classroom and clinical learning presents information in myriad ways and in different settings. Students should learn what they can from each situation and adapt their studying style as needed. Nursing knowledge, like patient care, is rarely straightforward and clear. Picking up on cues, emphasizing the correct element, answering the question being asked, and maintaining a person-centered focus that is individualized and nuanced requires practice and time. Students must give themselves the space and grace needed to learn and apply information in various situations. There is a reciprocity in learning that requires and inward and outward combination. Engagement with others and the environment is essential for shaping thinking and development of clinical judgment.

How well did you know this?
1
Not at all
2
3
4
5
Perfectly
31
Q

A nursing student tells the clinical instructor that their patient is fine and has “no complaints.” Which question by the faculty coaches the student to provide evidence that supports their assessments?

A. “Could you tell me how you validated this?”
B. “Do you think your patient feels free to share their concerns?”
C. “That’s good to hear. Tell me about the care you provided.”
D. “Please reassess the patient; they were admitted with a serious problem.”

A

A

RATIONALE:
a. The instructor is reminding the student that all data must be validated. Questioning the use of the word “fine” allows the nurse to determine if this is a social and reflexive response, and there may be another need the nurse can meet. Concluding that the patient does not trust the student is premature and is based on an invalidated inference. Saying “That’s good to hear” and asking the student to describe the care provided is incorrect because it accepts the invalidated inference. Telling the student to reassess the patient because they were admitted with a serious problem is incorrect because it is possible that the condition is resolving.

How well did you know this?
1
Not at all
2
3
4
5
Perfectly
32
Q

A nursery nurse notifies the nurse practitioner (NP) that a newborn has signs of jaundice. The NP performs a brief skin assessment, then orders a blood test for bilirubin levels. Which type of assessment has the NP performed?

A. Comprehensive
B. Initial
C. Time-lapsed
D. Quick priority

A

D

RATIONALE:
d. A quick priority assessment (QPA) is a short, focused assessment to obtain the most important information first. A comprehensive initial assessment is performed shortly after admission. The time-lapsed assessment is used to compare a patient’s current status to baseline data obtained earlier.

How well did you know this?
1
Not at all
2
3
4
5
Perfectly
33
Q

The nurse is admitting a pregnant patient to the hospital for treatment of pregnancy-induced hypertension. The patient asks the nurse, “Why are you doing a history and physical exam when the doctor just did one?” What statements will the nurse use to explain the primary purpose of the nursing assessment? Select all that apply.

A. “The nursing assessment will allow us to plan and deliver individualized, holistic nursing care that draws on your strengths.”
B. “It’s hospital policy. I know we ask a lot of questions, but I will try to make this quick.”
C. “As a nursing student, I need to develop assessment skills about your health status and need for nursing care.”
D. “This validates that your responses with the medical exam are consistent and that all our data are accurate.”
E. “I will check your health status and see what kind of nursing care you may need.”
F. “This is to determine the necessity for referring your nursing care needs to a health care provider.”

A

A, E, F

RATIONALE:
a, e, f. Medical assessments target data pointing to pathologic conditions, whereas nursing assessments focus on the patient’s responses to actual and potential health problems. The initial comprehensive nursing assessment results in baseline data that enable the nurse to make a judgment about a patient’s health status, the ability to manage their own health care and the need for nursing. It also helps nurses plan and deliver individualized, holistic nursing care that draws on the patient’s strengths and promotes optimum functioning, independence, and well-being, and enables the nurse to refer the finding(s) to the health care provider or collaborate with other health care professionals where indicated. Citing hospital policy or student learning is a secondary reason, and although it may be true that a nurse may need to develop assessment skills, it is not the main reason for a nursing history and assessment. The assessment augments the medical examination but is not performed to check its accuracy.

How well did you know this?
1
Not at all
2
3
4
5
Perfectly
34
Q

During shift report, a nurse says that a patient has no integumentary changes or skin care needs. During assessment, the nurse observes reddened areas over bony prominences. What action will the nurse take?

A. Correct the initial assessment form
B. Redo the initial assessment and document current findings
C. Conduct and document an emergency assessment
D. Perform and document a focused assessment of skin integrity

A

D

RATIONALE:
d. Perform a focused skin assessment for the new problem, documenting the current date. The initial assessment was entered in the permanent health record, correct at the time, and cannot legally be rewritten. An emergency assessment is performed for a life-threatening problem.

How well did you know this?
1
Not at all
2
3
4
5
Perfectly
35
Q

A nursing student is performing a nursing history for the first time. The student asks the primary nurse how anyone learns all the questions needed to get complete baseline data. What would be the nurse’s best reply?

A. “There’s a lot to learn at first, but once it becomes part of you, you just ask the same questions over and over in each situation until you can do it in your sleep!”
B. “You make the basic questions a part of you and apply critical thinking to modify them, to help you plan quality care.”
C. “It is really hard to learn how to do this well, as each history is different. I often feel like I’m starting fresh with each new patient.”
D. “Don’t worry about learning all of the questions to ask. Every facility has its own assessment form you must use.”

A

B

RATIONALE:
b. Once a nurse learns what constitutes the minimum data set, it can be adapted to each patient situation. It is not true that each assessment is the same even when using the same minimum data set, nor is it true that each assessment is uniquely different. Nurses committed to thoughtful, person-centered practice individualize their questions to each patient and situation. When using a standard facility assessment tool the nurse must still use critical thinking to individualize questions or follow up on patient information.

How well did you know this?
1
Not at all
2
3
4
5
Perfectly
36
Q

The nurse collects subjective and objective data during a patient assessment. When documenting, which data points will the nurse include as subjective data? Select all that apply.

A. Feeling nauseated
B. Edematous ankles
C. Feeling anxious about test results
D. Report of left arm tingling
E. Pain rated 7 on a scale of 1 to 10
F. Oral temperature of 101°F

A

A, C, D, E

RATIONALE:
a, c, d, e. Subjective data are information perceived only by the affected person. Examples of subjective data are feeling nauseated, anxious, tingling, and experiencing pain. Objective data are observable and measurable data that can be seen, heard, felt, or measured by someone other than the person experiencing them. Examples of objective data are fever or 101°F or edema.

How well did you know this?
1
Not at all
2
3
4
5
Perfectly
37
Q

When a nurse enters the patient’s room to begin a nursing history, the nurse notes the patient’s spouse is present. After greeting them, what action will the nurse take?

A. Thank the spouse for being present
B. Ask the spouse if they want to remain
C. Ask the spouse to leave
D. Ask the patient if they would like the spouse to stay

A

D

RATIONALE:
d. The patient has the right to privacy and to determine who will be present during the nursing history and exam. The nurse does not presume the patient’s preference, as the decision belongs to the patient, not their spouse.

How well did you know this?
1
Not at all
2
3
4
5
Perfectly
38
Q

A nurse is performing an initial comprehensive assessment of a patient admitted to a long-term care facility. The nurse begins the assessment by asking the patient, “How would you describe your health status and well-being?” and, “What do you do to keep yourself healthy?” These questions reflect what model for organizing data?

A. Maslow’s hierarchy of needs
B. Gordon’s functional health patterns
C. Human response patterns
D. Body system model

A

B

RATIONALE:
b. Gordon’s functional health patterns begin with the patient’s perception of health and well-being and progress to data about nutritional–metabolic patterns, elimination patterns, activity, sleep/rest, self-perception, role relationship, sexuality, coping, and values/beliefs. Maslow’s model is based on the human needs hierarchy. Human responses include exchanging, communicating, relating, valuing, choosing, moving, perceiving, knowing, and feeling. The body system model is based on the functioning of the major body systems.

How well did you know this?
1
Not at all
2
3
4
5
Perfectly
39
Q

The nurse notes a temperature of 102°F in a patient scheduled for surgery in 30 minutes. As the patient has been afebrile and asymptomatic until now, what action will the nurse take next?

A. Inform the charge nurse
B. Notify the surgeon
C. Reassess the temperature
D. Document the finding in the electronic health record

A

C

RATIONALE:
c. The nurse validates assessment findings that deviate from normal patterns or are unsupported by other data. Should the initial measurement be in error, it would have been premature to notify the charge nurse or the surgeon. The nurse must be sure that all data are accurate prior to documenting and reporting. If there is a question about accuracy, the data should be validated before documenting.

How well did you know this?
1
Not at all
2
3
4
5
Perfectly
40
Q

During a change-of-shift report, a nurse receives information that a patient admitted with hypertensive emergency has prescriptions for antihypertensive medications given at 8 AM and due at 8 PM. During the 8:00 PM assessment, the patient’s blood pressure is 90/60, and they report slight dizziness upon standing. After returning the patient to bed, what action will the nurse take?

Exhibit: Electronic health record, vital signs
8:00 AM 182/100
12:00 PM 168/98
4:00 PM 160/88

A. Record the BP in the electronic health record
B. Notify the health care provider
C. Administer the 8:00 PM medications
D. Place the patient flat in bed

A

B

RATIONALE:
b. When assessment findings reveal a critical change in the patient’s health status, the nurse reports the data (verbally) immediately. The nurse verifies the BP and notifies the health care provider, who may prescribe withholding blood pressure medications, assessment of orthostatic vital signs, among other actions. There is no indication the patient needs to lie flat at this time.

How well did you know this?
1
Not at all
2
3
4
5
Perfectly
41
Q

During an assessment, the nurse on a neurologic unit finds the patient confused to time and place but able to state their name. How will the nurse best record this in the electronic health record?

A. Is more confused than yesterday
B. States the year is 1975 and they are at a wedding
C. Disoriented to person, time, and place
D. Patient’s speech is garbled

A

B

RATIONALE:
b. While the patient is confused, it is most important to clearly describe the behavior for comparison to past and future behavior. Citing the actual year and events (orientation to time and place) provides context; “patient confused” is open to misinterpretation. Garbled speech refers to speech that is unclear; the patient may have difficulty with pronunciation or speak slowly, which is not necessarily reflective of confusion.

How well did you know this?
1
Not at all
2
3
4
5
Perfectly
42
Q

A nursing student is assigned to the emergency department (ED) to shadow the triage nurse. What activity will the student expect to perform?

A. Acute and emergency interventions
B. Daily care and assistance with ADLs
C. Assessment and prioritization of care
D. Care planning for return to home

A

C

RATIONALE:
c. The triage nurse screens patients to determine the extent and severity of their problems. They use highly specialized nursing knowledge and clinical reasoning and make clinical judgments to prioritize who must be seen immediately and who can wait. Patients in the ED are stabilized and transferred to the appropriate level of care; therefore, daily care, assistance with ADLs, planning for return home, and providing interventions are not part of the triage nurse’s role. Should the patient need emergency interventions, the triage nurse moves the patient to the appropriate area in the ED.

How well did you know this?
1
Not at all
2
3
4
5
Perfectly
43
Q

The nurse is assigned to care for a group of patients. Which patient will the nurse assess first?

A. Postoperative patient reporting pain 4/10
B. Individual with pneumonia whose WBCs are now 7,000
C. Adolescent with a burn to the face who is going home tomorrow
D. Patient’s pulse oximetry reading 89%, as reported by AP

A

D

RATIONALE:
d. The nurse uses Maslow’s hierarchy of needs to prioritize assessing the patient with hypoxemia, manifested by a pulse oximetry reading of 89%. The postoperative patient is reporting moderate pain; the patient with pneumonia has normal WBCs; the adolescent patient with a burn to the face is stable for discharge. These patients can be seen as soon as possible.

How well did you know this?
1
Not at all
2
3
4
5
Perfectly
44
Q

The nurse assessing a patient plans to use the OLD CARTS mnemonic to organize their questions. What questions will the nurse include in the assessment? Select all that apply.

A. “Can you tell me when the problem began”?
B. “Where were you sitting when this started?”
C. “Have your symptoms stopped and/or started again?”
D. “Would you describe your pain as sharp, dull or burning?”
E. “What do you believe has caused this problem?”

A

A,C,D

RATIONALE:
a, c, d. The OLD CARTS mnemonic refers to Onset: “When did your symptom(s) begin”?. Location: “Where is the symptom”?. Duration: “Is it episodic or constant?” or “How long does it last”?. Characteristics: “How would you describe it?.” Alleviating and Aggravating factors: “What makes it better or worse?.” Relieving factors: “What makes it better?.” Treatments: “Have you tried anything to make it better?.” Severity: “On a scale of 1–10, with 1 being the lower number, how serious is the symptom?.”

How well did you know this?
1
Not at all
2
3
4
5
Perfectly
45
Q

A nurse in the emergency department is assessing a young adult who has cognitive disability and is reporting severe abdominal pain. The patient is accompanied by the director of the group home where they live. When collecting data from this patient, which action reflects best practice?

A. Ask the assessment questions of the director.
B. Wait for the young adult’s parents to arrive before performing the assessment.
C. Ask the young adult questions and validate with the adult present.
D. Perform the physical assessment, then the intake interview when the family arrives.

A

C

RATIONALE:
c. Children and people with decreased mental capacity or impaired verbal ability should be encouraged to respond to interview questions as best as they can. This communicates support of the patient’s autonomy, expression of their needs, and respect for their abilities. The information is then validated with family members or guardians as appropriate.

How well did you know this?
1
Not at all
2
3
4
5
Perfectly
46
Q

A nursing student on the surgical unit is assigned to perform a review of systems using the head-to-toe format on a patient admitted for a fractured femur. Using this format, what system will the student assess first?

A. Genitourinary
B. Neurologic
C. Respiratory
D. Musculoskeletal

A

B

RATIONALE:
b. The nursing physical assessment involves the examination of all body systems in a systematic manner, commonly using a head-to-toe format called the review of systems (ROS). This assessment begins at the top of the body with the neurologic system and moves downward.

How well did you know this?
1
Not at all
2
3
4
5
Perfectly
47
Q

A registered nurse is formulating nursing diagnoses for a patient with multiple fractures. Which actions does the nurse take during this step of the nursing process? Select all that apply.

A. Conducting a nursing interview to collect patient data
B. Analyzing data collected in the nursing assessment
C. Developing a care plan for the patient
D. Pointing out the patient’s strengths
E. Assessing the patient’s mental status
F. Identifying community resources to help the family cope

A

B,D,F

RATIONALE:
b, d, f. Diagnosing includes identifying actual or potential health problems for individuals, groups, or communities; identifying factors that contribute to or cause health problems (etiologies); and identifying resources or strengths the individual, group, or community can draw on to prevent or resolve problems. The nurse assesses and collects patient data in the assessment step and develops the care plan during the planning phase of the nursing process.

How well did you know this?
1
Not at all
2
3
4
5
Perfectly
48
Q

A nurse is caring for a patient who presents with dyspnea, tachypnea, productive cough, fever, and low oxygen saturation. When developing the nursing care plan, which health problems might the nurse identify for this patient? Select all that apply.

A. Bronchial pneumonia
B. Impaired gas exchange
C. Impaired Respiratory System Function
D. Altered breathing pattern
E. Impaired Thermoregulation

A

B,C,D,E

RATIONALE:
b, c, d, e. Nursing diagnoses are actual or potential health problems that can be prevented or resolved by independent or interdependent nursing interventions. These include Impaired gas exchange, supported by low oxygen saturation; Impaired Respiratory System Function; Altered breathing pattern, supported by dyspnea and tachypnea; and Impaired Thermoregulation. Pneumonia is a medical diagnosis.

How well did you know this?
1
Not at all
2
3
4
5
Perfectly
49
Q

After assessing a patient recovering from a stroke in a rehabilitation facility, the nurse’s initial analysis suggests a potential health problem of situational low self-esteem. How will the nurse record the problem when they believe more data are needed?

A. No problem
B. Possible problem
C. Actual nursing diagnosis
D. Clinical problem other than nursing

A

B

RATIONALE:
b. When a possible problem exists, the nurse must collect more data to confirm or disprove the suspected problem. The conclusion, “no problem,” indicates no nursing response is required. When an actual problem is identified, the nurse continues using the steps of the nursing process by planning, goal setting, implementing, and evaluating care to resolve the problem. A clinical problem other than nursing diagnosis requires a collaborative approach with the appropriate health care professionals.

How well did you know this?
1
Not at all
2
3
4
5
Perfectly
50
Q

When caring for a patient who sustained a spinal cord injury, the nurse formulates the health problem:
Impaired Tissue Integrity
Etiology: sensory and motor deficit
Signs and symptoms: difficulty turning, reddened areas on heels and sacrum
Which phrase gives direction to the underlying cause of the problem?

A. Impaired Tissue Integrity
B. Sensory and motor deficit
C. Signs and symptoms
D. Reddened areas of skin on the heels and back

A

B

RATIONALE:
b. The etiology, sensory and motor deficits, identifies the contributing or causative factors of the problem. The problem, “Impaired Tissue Integrity: Impaired Skin Integrity,” states the undesirable health condition, life processes, or human response. The phrase, “Signs and symptoms: non-blanchable reddened areas on heels and back,” contains the defining characteristics of the problem.

How well did you know this?
1
Not at all
2
3
4
5
Perfectly
51
Q

A nurse is caring for a patient who refuses to look at or care for a new colostomy. The patient states, “I don’t care what I look like anymore. I’m not washing up, let alone touching or changing this bag!” The nurse formulates the health problem: Difficulty Coping: Impaired Acceptance of Health Status, reflecting which type of health problem?

A. Collaborative
B. Interdisciplinary
C. Medical
D. Nursing

A

D

RATIONALE:
d. Difficulty Coping: Impaired Acceptance of Health Status is a nursing problem, falling within the scope of independent nursing practice. Collaborative and interdisciplinary problems are resolved through a teamwork approach with other health care professionals. A medical problem is a traumatic or disease condition validated by medical diagnostic studies.

How well did you know this?
1
Not at all
2
3
4
5
Perfectly
52
Q

A nursing student obtains a blood pressure reading of 148/100. To determine the significance of this reading, what action will the nurse take first?

A. Comparing this reading to standards and trends in the medical record
B. Checking the taxonomy of nursing diagnoses for a pertinent label
C. Checking a medical text for the signs and symptoms of high blood pressure
D. Consulting with experienced nurse colleagues

A

A

RATIONALE:
a. A standard, or a norm, is a generally accepted rule, measure, pattern, or model to which data can be compared within the same class or category. When interpreting the significance of a patient’s blood pressure reading, the nurse uses normative values for the patient’s age group, race, and illness category and compares these to the patient’s recent results. Identifying the reason for deviation from a norm gives direction to the etiology of a health problem (e.g., insufficient knowledge, nutrition, stress, and coping, or other).

How well did you know this?
1
Not at all
2
3
4
5
Perfectly
53
Q

A nursing student tells the primary nurse that their patient has not had a bowel movement for 2 days and suggests adding the health problem “Constipation” to the care plan. How would the nurse best respond?

A. “Did you assess the patient’s usual bowel patterns and appearance of the last stool?”
B. “This early diagnosis will help us manage the problem before it becomes severe.”
C. “Have you determined if this is an actual or a possible diagnosis?”
D. “This condition requires a medical diagnosis.”

A

A

RATIONALE:
a. Patient health problems are derived from clusters of related data and patterns, rather than a single cue. The nurse determines if this is the patient’s usual bowel pattern, or whether an underlying reason exists for the lack of a bowel movement. Constipation is a health problem the nurse can resolve with independent or interdependent nursing actions.

How well did you know this?
1
Not at all
2
3
4
5
Perfectly
54
Q

A nurse is caring for a patient who has been admitted the second time this month for hypertensive emergency. The care plan contains the health problem:
Nonadherence
Etiology: lack of knowledge of purpose of medications
Signs and symptoms: BP, 220/112; readmitted for hypertensive crisis after 2 weeks
When meeting the patient, which action will the nurse take first?

A. Teach the patient that nonadherence may lead to stroke and heart disease
B. Discuss what will motivate the patient to adhere to the medication regimen
C. Explain that these medications are essential to their health and illness prevention
D. Determine the patient’s knowledge about the medications and their side effects

A

D

RATIONALE:
d. Using the nursing process, the nurse first assesses the patient’s knowledge base; this also confirms the accuracy of the problem statement. Problem statements with unclear etiologies may lead to inappropriate, erroneous, or unhelpful interventions. If the patient has difficulty affording medications or is experiencing side effects, a collaborative problem can be resolved jointly by the nurse, social worker, and health care provider.

How well did you know this?
1
Not at all
2
3
4
5
Perfectly
55
Q

A nurse in the psychiatric clinic is developing a problem list for a patient. What statement best reflects a correctly written, two-part problem?

A. Difficulty Coping: Impaired Family Coping
Etiology: inability to maintain marriage
B. Difficulty Coping: Impaired Acceptance of Health Status
Etiology: anger management issues
C. Impaired Cognition: Distorted Thought Process
Etiology: psychosis as evidence by hallucinations
D. Impaired Cognition: Decisional Conflict
Etiology: placement of parent in a long-term care facility

A

D

RATIONALE:
d. A correctly written two-part problem statement includes the health problem and the etiology or cause. The problem statement and etiology should avoid signs and symptoms, medical diagnoses, and something that cannot be changed. Inability to maintain marriage and anger issues do not identify the underlying cause of the problem and may themselves reflect the true problem. Psychosis is a medical diagnosis, which should not be used to support a patient problem.

How well did you know this?
1
Not at all
2
3
4
5
Perfectly
56
Q

A nurse is developing a problem list for a care plan. Which reflects a correctly written three-part problem statement? Select all that apply.

A. Difficulty Coping: Impaired Family Coping:
Etiology: lack of knowledge about tube feeding
Signs and symptoms: child needing tube feeding discharged to home
B. Impaired Nutritional Status: Impaired Nutritional Intake
Etiology: striving for perfect weight, wishes to excel in gymnastics
Signs and symptoms: 20-lb weight loss in 1 month
C. Need to learn how to care for child on ventilator at home
Etiology: discharge of child after 3-month hospital stay
Signs and symptoms: repeated comments, “I know I’ll harm her because I’m not a nurse.” and “I can’t do medical things.”
D. Impaired Spiritual Status
Etiology: inability to accept diagnosis of terminal illness
Signs and symptoms: comments such as, “I don’t deserve this”; “I’ve tried to live my life well”; and “How could God make me suffer this way?”
E. Impaired Tissue Integrity: Impaired Skin Integrity
Etiology: failure of home health aides to turn patient every 2 hours
Signs and symptoms: stage 3 pressure wound on sacrum.

A

D

RATIONALE:
d. Correctly written problem statements contain a problem the nurse can treat with independent or interdependent interventions, a clearly stated etiology or cause of the problem, and supporting signs and symptoms. Option (a) may be more easily resolved with the problem statement, knowledge deficiency. Option (a) further states the tube feeding is the underlying cause of the problem; it is a factor that cannot be changed. Option (c) is written in terms of needs and not an unhealthy response. Option (e), while written in three parts, places blame or implies negligence, which is legally inadvisable and should be avoided. A clear etiology is not stated in option (e), impeding direction for appropriate interventions or outcomes.

How well did you know this?
1
Not at all
2
3
4
5
Perfectly
57
Q

A nurse is caring for a patient recovering from a stroke that paralyzed the dominant arm. The nursing assistant reports that the patient was unable to bathe, comb their hair, or brush their teeth. Which health problem should the nurse add to the care plan?

A. Lack of motivation to complete self-care activities
B. Risk for: Activities of Daily Living Deficit
C. ADL deficit: impaired dressing and grooming
D. Impaired musculoskeletal system function: paralysis

A

C

RATIONALE:
c. The nurse clusters the data that demonstrates the patient’s (actual) inability to perform bathing and grooming. For that reason, a “potential problem” or “risk” is not appropriate. There is no evidence the patient lacks motivation, and paralysis is not a problem the nurse can resolve.

How well did you know this?
1
Not at all
2
3
4
5
Perfectly
58
Q

A nurse is caring for a patient who had abdominal surgery yesterday. The nurse observes the patient guarding the area with hands and a pillow, refusing to move, and grimacing. What information does the nurse use to formulate the health problem statement?

A. Symptoms
B. Diagnostic statement
C. Etiology
D. Cue

A

D

RATIONALE:
d. A cue denotes significant data or “red flags,” that, when occurring in a pattern or cluster, point to the existence of a health problem.

How well did you know this?
1
Not at all
2
3
4
5
Perfectly
59
Q

When developing the admission care plan for a patient with multiple sclerosis and quadriplegia, the nurse formulates the patient problem: Impaired Tissue Integrity: Impaired Skin Integrity. What action will the nurse take next?

A. Elevate the patient’s heels off the bed using a pillow
B. Develop a goal that the patient will consume protein at each meal
C. Delegate assessment of the skin on the patient’s back to the AP
D. Teach the patient to turn themselves in bed every hour

A

B

RATIONALE:
b. After the health problem is developed, the nurse begins the planning phase of the nursing process, which includes goal development. Elevating the heels and teaching the patient to turn are interventions used during the implementation phase. Delegating an assessment to the AP is an incorrect activity; assessment falls within the role and scope of practice of the professional nurse. In addition, the nurse must perform the skin assessment to develop the problem and plan care.

How well did you know this?
1
Not at all
2
3
4
5
Perfectly
60
Q

A nurse has performed an admission assessment on a patient. What step does the nurse perform after clustering the data?

A. Developing interventions
B. Nursing judgments
C. Diagnosing and analyzing
D. Concept mapping

A

C

RATIONALE:
c. After clustering the data, the nurse analyzes the data and formulates a nursing diagnosis. Interventions are based on and developed after goal setting. Nursing judgments are outcomes based on critical thinking and clinical reasoning. A concept map is a diagram or pictorial representation of the (student) nurse’s understanding of the interactions and relationships of the patient’s problems and plan of care.

How well did you know this?
1
Not at all
2
3
4
5
Perfectly
61
Q

A nurse notices a patient crying after meeting with the health care provider. Prior to formulating a health problem of difficulty coping, the nurse seeks to further support the problem by gathering which data?

A. Abnormal vital signs
B. Underlying cause of the tears
C. Admitting diagnosis
D. Patient’s support system

A

B

RATIONALE:
b. The nurse continues gathering data, determining the presence of a problem of grief, impaired coping, etc., by determining the underlying cause of the tears. If the patient received news that a biopsy was free from cancer, perhaps no problem exists. If the patient was told they have a terminal illness, the nurse can continue to gather data and plan to support the patient’s physical and emotional needs.

How well did you know this?
1
Not at all
2
3
4
5
Perfectly
62
Q

A nurse is planning care for a patient admitted to the hospital for treatment of a drug overdose. What actions will the nurse take during the outcome identification and planning step of the nursing process? Select all that apply.

A. Formulating nursing diagnoses
B. Identifying expected patient outcomes
C. Selecting evidence-based nursing interventions
D. Explaining the nursing care plan to the patient
E. Assessing the patient’s mental status
F. Evaluating the patient’s outcome achievement

A

B,C,D

RATIONALE:
b, c, d. During the outcome identification and planning step of the nursing process, the nurse, patient, and family collaborate to establish priorities and identify and write expected patient outcomes. The nurse selects evidence-based nursing interventions, and communicates the care plan. These steps may overlap; however, formulating and validating nursing diagnoses are typically performed during the diagnosing step. Assessing mental status is part of the assessment step, and evaluating patient outcomes occurs during the evaluation step of the nursing process.

How well did you know this?
1
Not at all
2
3
4
5
Perfectly
63
Q

Nurses on a hospital unit work to improve staff communication, as outlined in The Joint Commission’s National Patient Safety Goals. What process will best provide for continuity of the plan of care?

A. Checking two patient identifiers, such as name and date of birth, prior to administering medications
B. Ensuring two nurses check doses of high-risk medications such as anticoagulants or insulin
C. Giving handoff report in the patients’ rooms to update the next nurse on the plan of care
D. Obtain a patient sitter for a confused individual who has fallen trying to get out of bed

A

C

RATIONALE:
c. One of the published standards and requirements for accreditation and certification required by The Joint Commission is to “improve staff communication.” Communicating the plan of care with the patient and oncoming and off going nurses meets this goal. Using patient identifiers relates to the goal of safely identifying patients, checking high-risk medications relates to decreasing medication errors, and obtaining a patient sitter relates to general safety and fall reduction.

How well did you know this?
1
Not at all
2
3
4
5
Perfectly
64
Q

A nurse on a mother–baby unit engages in informal planning while providing ongoing nursing care. What actions are included in this type of planning? Select all that apply.

A. Sitting down with a patient and prioritizing existing diagnoses
B. Assessing a woman for postpartum depression during patient education
C. Planning interventions for a patient with a risk for bleeding
D. Taking time to speak with a new mother who just received bad news
E. Reassessing a patient who reports their pain medication is not working
F. Coordinating home care for a patient being discharged later today

A

B, D,E

RATIONALE:
b, d, e. Informal planning is a link between identifying a patient’s strength or problem and providing an appropriate nursing response, often while rearranging priorities. Examples of this include the nurse integrating assessment for postpartum depression during patient care, providing a therapeutic presence for a patient who received bad news, or reassessing a patient for pain during rounding. Formal planning involves prioritizing diagnoses, formally planning interventions, and coordinating the home care of a patient being discharged.

How well did you know this?
1
Not at all
2
3
4
5
Perfectly
65
Q

To plan the day, a nurse is prioritizing patient diagnoses according to Maslow’s hierarchy of human needs. What patient problem will the nurse address first?

A. Altered body image perception
B. Impaired gas exchange
C. Grief
D. Situational low self-esteem

A

B

RATIONALE:
b. Because basic needs must be met before a person can focus on higher ones, Maslow’s hierarchy of needs sets the priorities as: (1) physiologic needs, (2) safety needs, (3) love and belonging needs, (4) self-esteem needs, and (5) self-actualization needs. Answer (b) is an example of a physiologic need, (a and d) are examples of a self-esteem need, and (c) is an example of a love and belonging need.

How well did you know this?
1
Not at all
2
3
4
5
Perfectly
66
Q

Nurses on an oncology unit plan to adopt use of critical pathways for patients receiving chemotherapy. What positive features of this system will the nurses anticipate? Select all that apply.

A. Accessible computerized practice standards, easily individualized for patients
B. Binary decision tree for stepwise assessment and intervention
C. Ability to measures the cause-and-effect relationship between pathway and patient outcomes
D. Research-based practice recommendations that may or may not have been tested in clinical practice
E. Preprinted provider prescriptions, using standards validated through research, to streamline care
F. Outcomes with suggested time frames for achievement

A

A,C,F

RATIONALE:
a, c, f. Critical pathways represent a sequential, interdisciplinary, minimal practice standard for a specific patient population, that provide flexibility to alter care to meet individualized patient needs. They provide the ability to measure a cause-and-effect relationship between pathway and patient outcomes. An algorithm is a binary decision tree that guides stepwise assessment and intervention with intense specificity and no provider flexibility. Guidelines are broad, research-based practice recommendations that may or may not have been tested in clinical practice, and an order set is a preprinted provider order used to expedite the order process after a practice standard has been validated through analytical research.

How well did you know this?
1
Not at all
2
3
4
5
Perfectly
67
Q

A nurse is developing outcomes in the affective domain for a patient with a foot ulcer related to diabetes. Which outcome best addresses this domain?

A. Within 1 day after teaching, the patient will list three benefits of continuing to apply moist compresses to foot ulcer after discharge.
B. By 6/12/25, the patient will correctly demonstrate application of wet-to-dry dressing on the foot ulcer.
C. By 6/19/25, the patient’s pressure ulcer will decrease in size from 3 to 2.5 inches.
D. By 6/12/25, the patient will verbalize they value their health sufficiently to control diabetes and prevent recurrence of diabetic ulcers.

A

D

RATIONALE:
d. Affective outcomes describe changes in patient values, beliefs, and attitudes. Cognitive outcomes (a) describe increases in patient knowledge or intellectual behaviors; psychomotor outcomes (b) describe the patient’s achievement of new skills; and (c) is an outcome describing a physical change in the patient.

How well did you know this?
1
Not at all
2
3
4
5
Perfectly
68
Q

A nurse is developing a clinical outcome for a patient who is an avid runner and is recovering from a stroke resulting in right-sided paresis. Which clinical outcome is most appropriate to include in the care plan?

A. After receiving 3 weeks of physical therapy, patient will demonstrate improved movement on the right side of her body.
B. By 8/15/25, patient will be able to use right arm to dress, comb hair, and feed herself.
C. Following physical therapy, patient will begin to gradually participate in walking/running events.
D. By 8/15/25, patient will verbalize feeling sufficiently prepared to participate in running events.

A

B

RATIONALE:
b. Clinical outcomes describe the expected status of health issues at certain points in time, after treatment is complete. Functional outcomes (b) describe the person’s ability to function in relation to the desired usual activities. Quality-of-life outcomes (c) focus on key factors that affect someone’s ability to enjoy life and achieve personal goals. Affective outcomes (d) describe changes in patient values, beliefs, and attitudes.

How well did you know this?
1
Not at all
2
3
4
5
Perfectly
69
Q

A nurse is caring for a patient with dehydration who has a prescription to encourage oral fluids. Which outcome statement will best direct nursing interventions?

A. Offer patient 60 mL of fluid every 2 hours while awake.
B. During the next 24-hour period, patient’s fluid intake will total at least 2,000 mL.
C. Teach the patient the importance of drinking enough fluids to prevent dehydration by 1/15/25.
D. At the next visit on 12/23/24, patient will know to drink at least 3 L of water per day.

A

B

RATIONALE:
b. The outcomes in (a) and (c) make the error of expressing the patient goal as a nursing intervention. Incorrect: “Offer the patient 60 mL fluid every 2 hours while awake.” Correct: “The patient will drink 60 mL fluid every 2 hours while awake, beginning 1/3/25.” The outcome in (d) makes the error of using verbs that are not observable and measurable. Verbs to be avoided when writing outcomes include “know,” “understand,” “learn,” and “become aware.”b. The outcomes in (a) and (c) make the error of expressing the patient goal as a nursing intervention. Incorrect: “Offer the patient 60 mL fluid every 2 hours while awake.” Correct: “The patient will drink 60 mL fluid every 2 hours while awake, beginning 1/3/25.” The outcome in (d) makes the error of using verbs that are not observable and measurable. Verbs to be avoided when writing outcomes include “know,” “understand,” “learn,” and “become aware.”

How well did you know this?
1
Not at all
2
3
4
5
Perfectly
70
Q

A nurse is writing outcomes for a patient admitted with a cardiac condition causing fluid overload and edema. Which reflects an appropriately worded outcome?

A. Offer to elevate the patient’s legs on a stool while out of bed
B. Patient will restrict fluids to 1,500 mL per 24-hour period
C. Monitor the patient’s intake and output
D. Weigh the patient each morning prior to breakfast

A

B

RATIONALE:
b. The terms goal, objective, and outcome are often used interchangeably to refer to the expected conclusion to the patient’s health problem or expectation. Nurses use the phrase expected outcomes to refer to the more specific, observable, and measurable changes. Options a, c, and d are stated as interventions, rather than outcomes.

How well did you know this?
1
Not at all
2
3
4
5
Perfectly
71
Q

A nursing student is presenting their concept map care plan for a patient with sickle cell anemia in post-conference. How does the student best describe the “concepts” that are being diagrammed in the plan?

A. Protocols for treating the patient’s medical problem
B. Evidence-based treatment guidelines
C. Synthesis of the patient’s problems and treatment
D. Clinical pathways reflecting evidence-based treatment for sickle cell anemia

A

C

RATIONALE:
c. A concept map care plan is a diagram synthesizing patient problems and interventions. The nurse’s ideas about patient problems and treatments are the “concepts” that are diagrammed. These maps are used to organize patient data, analyze relationships in the data, and provide a holistic view of the patient’s situation. Answers (a) and (b) are incomplete because the concepts being diagrammed may include protocols and standardized treatment guidelines but the patient problems are also diagrammed concepts. Clinical pathways are tools used in case management to communicate the standardized, interdisciplinary care plan for patients.

72
Q

A nurse is updating the plan of care with nurse-initiated interventions. Which intervention is appropriate to include?

A. Administering acetaminophen for a headache
B. Offering emotional support to a patient
C. Consulting with a physical therapist
D. Attending a team meeting for care planning

A

B

RATIONALE:
b. A nurse-initiated intervention is related to the nursing diagnosis and projected outcome. It is an autonomous action based on scientific rationale. The physician or health care provider uses a physician-initiated interventions or order in response to the medical diagnosis: nurses execute these interventions safely and effectively. Collaborative interventions are initiated by other providers including pharmacists, respiratory therapists, or physician assistants.

73
Q

A school nurse determines that a student who has lost weight is at risk for an eating disorder and would benefit from a nutritional assessment. What action will the nurse take?

A. Perform a focused nutritional assessment
B. Seek direction from the student’s health care provider
C. Suggest the student visit the nurse-run clinic
D. Request a consultation with a nutritionist

A

A

RATIONALE:
a. Performing a focused assessment is an independent nurse-initiated intervention, not requiring a prescription from or intervention by the health care provider, advanced practice nurse, or nutritionist.

74
Q

A nurse is caring for a group of patients. Which actions are appropriate to include in the implementation phase of care? Select all that apply.

A. Changing the dressings on a burn victim’s arm
B. Assessing a patient’s nutritional intake
C. Formulating a nursing diagnosis for a patient with epilepsy
D. Turning a patient in bed every 2 hours to prevent pressure injuries
E. Checking a patient’s insurance coverage at the initial interview
F. Determining availability of community resources for a patient with dementia

A

A,D,F

RATIONALE:
a, d, f. During the implementing step of the nursing process, nursing actions that were formulated during the planning process are carried out. The purpose of the implementation phase is to assist the patient in achieving valued health outcomes, for example promote health, prevent disease and illness, restore health, and facilitate coping with altered functioning. Assessing a patient’s nutritional status or insurance coverage occurs in the assessment step, and formulating nursing diagnoses occurs in the diagnosing/analyzing step.

75
Q

Nurses use the Nursing Interventions Classification (NIC) Taxonomy structure as a resource to plan nursing care for patients. What information is found in this structure?

A. Case studies illustrating a complete set of activities that a nurse performs to carry out nursing interventions
B. Nursing interventions, each with a label, a definition, and a set of activities that a nurse performs to carry it out, with a short list of background readings
C. Complete list of nursing diagnoses, outcomes, and related nursing activities for each nursing intervention
D. Complete list of reimbursable charges for each nursing intervention

A

B

RATIONALE:
b. The Nursing Interventions Classification (NIC) Taxonomy lists nursing interventions, each with a label, a definition, a set of activities that a nurse performs to carry it out, and a short list of background readings. It does not contain case studies, diagnoses, or charges.

76
Q

A nurse works in a long-term care facility where standing orders are in place for influenza vaccines for all residents. What is the nurse’s priority, when carrying out the prescriptions?

A. Assessing whether the patient previously received the vaccine
B. Refusing to give the vaccine without a written prescription
C. Determining if the standing orders are inappropriate for their unit
D. Calling the nursing supervisor to determine if this is a permitted action

A

A

RATIONALE:
a. Standard orders empower the nurse to initiate actions ordinarily requiring the order, prescription or supervision of a health care provider. The nurse first assesses whether the patient has already received the vaccine. The standing order is a valid prescription given to cover common, recurring actions the nurse can use when indicated.

77
Q

A nurse performs nurse-initiated nursing actions when caring for patients in a skilled nursing facility. Which reflect these types of actions? Select all that apply.

A. Administering an antibiotic to a patient with pneumonia
B. Consulting with a psychiatrist for a patient who misuses opiates
C. Checking the skin of bedridden patients for skin breakdown
D. Ordering a kosher meal for an orthodox Jewish patient
E. Recording a patient’s intake and output
F. Preparing a patient for surgery according to facility protocol

A

C,D,F

RATIONALE:
c, d, f. Nurse-initiated interventions, or independent nursing actions, include nurse-prescribed interventions resulting from their assessment of patient’s actual or potential health problems. Protocols and standard orders empower the nurse to initiate actions that ordinarily require the order or supervision of a health care provider. Consulting with a psychiatrist is a collaborative intervention.

78
Q

A nurse enters the patient’s room to perform pin-site care for a patient wearing a halo vest to stabilize the cervical spine. What action will the nurse take first?

A. Administer pain medication
B. Reassess the patient
C. Prepare the equipment
D. Explain the procedure to the patient

A

B

RATIONALE:
b. Before implementing any nursing action, the nurse returns to the first step of the nursing process, reassessing whether the action is still needed. Then the nurse may collect the equipment, explain the procedure, and, if necessary, administer pain medications.

79
Q

The charge nurse tells a nursing student to change a surgical dressing while they take care of other patients. The student has not changed dressings before and does not feel confident performing the procedure. What action should the student take?

A. Tell the charge nurse that they lack the technical competencies to change the dressing independently
B. Assemble the equipment for the procedure and follow the steps in the procedure manual
C. Ask another student nurse to work collaboratively with them to change the dressing
D. Tell the clinical instructor they have not had experience with the delegated task

A

A

RATIONALE:
a. Student nurses should notify their nursing instructor or nurse preceptor if they believe they lack any competencies needed to safely implement the care plan. Once educated and technically prepared, the nursing student may perform a dressing change.

80
Q

A nurse develops a care plan for an adolescent patient who gave birth to a premature infant. When presented with the collaborative care plan, including home health care visits, the patient states, “We will be fine on our own. I don’t need any more care.” What is the nurse’s best response?

A. “You know your personal situation better than I do; I will respect your wishes.”
B. “If you don’t accept these services, your baby’s health will suffer.”
C. “Let’s take a look at the plan again and see if we can adjust it to fit your needs.”
D. “I’m going to assign your case to a social worker who can explain the services better.”

A

C

RATIONALE:
c. When a patient rejects the care plan, the nurse works to identify the underlying barriers. If the nurse determines that the care plan is adequate, the nurse works with the patient to formulate mutually developed goals and interventions.

81
Q

An RN working on a hospital unit frequently delegates patient care to assistive personnel (AP). Which activities are appropriate for the nurse to safely delegate? Select all that apply.

A. Performing patient assessments
B. Making patient beds
C. Giving patients bed baths
D. Administering oral medications
E. Ambulating patients
F. Assisting patients with meals

A

B,C,E,F

RATIONALE:
b, c, e, f. AP assist the RN to provide care as delegated by and under their supervision. Typical tasks delegated include actions for stable patients (e.g., vital signs hygiene, bed-making, ambulating patients, and helping to feed patients). Performing the initial patient assessment and administering medications are the responsibility of the RN.

82
Q

A nursing student is prioritizing interventions for a patient with diabetes who needs diagnostic testing, dressing changes, meal planning and counseling, and assistance with hygiene. The patient states, “I must have my hair washed before I can do anything else; I’m ashamed of the way I look.” How will the student best prioritize this patient’s care?

A. Explain to the patient that there is not enough time to wash their hair today because of the busy schedule
B. Schedule the testing and meal planning first and complete hygiene as time permits
C. Perform the dressing changes first, schedule the testing and counseling, and complete hygiene last
D. Wash the patient’s hair and perform hygiene, schedule testing and counseling, then change the dressing

A

D

RATIONALE:
d. When time constraints and safety permit, priorities identified by the patient as most important are completed first. Washing the patient’s hair and assisting with hygiene put the patient first, setting the tone for an effective nurse–patient partnership.

83
Q

A nurse is caring for a patient with a painful, non-healing surgical wound. The patient does not request pain medication because they do not want to be a burden. What actions will the nurse implement to improve pain relief? Select all that apply.

A. Reestablishing the pain level the patient finds acceptable as the pain management goal
B. Obtaining a dry-erase board to remind the patient of the plan of care
C. Assessing the patient’s pain and offering analgesia during hourly rounding
D. Placing the analgesic underneath other medications and quickly handing it to the patient
E. Asking the family members to speak to the patient about pain relief

A

A,D,C

RATIONALE:
a, b, c. The nurse reassesses the patient’s knowledge and acceptance of the plan of care, including the level of pain the patient finds acceptable. Using a dry-erase board and hourly rounding further communicate and reinforce the care plan. The nurse develops a compassionate and trusting relationship with the patient; the nurse and patient mutually determine the plan for pain management, not their family.

84
Q

A nursing unit has adopted use of a care bundle for insertion of central venous catheters. During the procedure, which action by a nurse requires the charge nurse to intervene?

A. They discard the sterile drapes in the insertion kit.
B. The primary nurse reminds everyone in the room to wear a mask.
C. The team includes every item in the bundle during the procedure.
D. The nursing student states using the bundle improves patient outcomes.

A

A

RATIONALE:
a. Care bundles are sets of evidence-based interventions that, when performed together and consistently, improve the process of care and patient outcomes. Discarding sterile drapes from the insertion kit circumvents this process.

85
Q

A university student works with the student health nurse to develop a weight loss plan that includes increasing activity and avoiding empty calories. At the next session, the student has lost 1 lb instead of the projected 5 lb. What action will the nurse take next?

A. Congratulate the student and continue the care plan
B. Terminate the care plan since it is not working
C. Give the student more time to reach the targeted outcome
D. Modify the plan after discussing possible reasons for partial success

A

D

RATIONALE:
d. Since the student has only partially met her outcome, the nurse should first explore the factors making it difficult for her to reach her outcome and then modify the care plan. As the plan is not completely working as written, continuing without further assessment is contraindicated. It is premature to terminate the care plan before the outcome is met. The student may need additional support and time to reach the outcome.

86
Q

A visiting nurse is following up with a patient who was given a prescription for a diuretic and told to chart her weight daily. The patient’s weight has increased 5 lb since the nurse’s last visit. What actions will the nurse take first?

A. Explain to the patient that it is clear she is not adhering to her prescription and the health care provider will be notified
B. Document the 5-lb weight gain and ask the patient about sodium intake and medication side effects
C. Terminate the plan of care while determining the cause for the weight gain
D. Encourage the patient to continue the prescription and return in 1 week

A

B

RATIONALE:
b. The nurse documents the goal has not yet been achieved and also suspects the patient has not adhered to the prescription, perhaps due to frequent urination or other side effects. The nurse further assesses the patient’s understanding of the medication’s purpose and effects, understanding of the disease process and complications.

87
Q

During orientation to the critical care unit, a nurse learns that staff follow existing clinical practice guidelines, also called standards, for patient care. Which activities does the nurse expect to be included in these guidelines? Select all that apply.

A. Monitoring vital signs and pulse oximetry every hour
B. Using intuition to troubleshoot patient problems
C. Repositioning a patient on bed rest every 2 hours
D. Becoming a nurse mentor to a student nurse
E. Administering pain medication prescribed by the health care provider
F. Becoming involved in community nursing events

A

A,C,E

RATIONALE:
a, c, e. Standards are the levels of performance accepted and expected by the nursing staff or other health care team members. They are established by authority, custom, or consent. Standards would include monitoring patient status every hour, repositioning a patient on bed rest every 2 hours, and administering pain medication prescribed by the health care provider. Using intuition to troubleshoot patient problems, becoming a nurse mentor to a student nurse, and becoming involved in community nursing events are not included in patient care standards.

88
Q

After reviewing the admission SBAR and plan of care, the nurse begins to evaluate patient outcomes. Which statement reflects a clear evaluation of the patient’s primary problem?

Electronic health record (EHR)
8:00 AM Admission note
S. Patient with profound wheezing, tachycardia, and anxiety
B. Patient has history of asthma, for which she regularly uses inhalers and carries a rescue inhaler
A. Pulse oximetry 89%, cyanosis of lips, dyspnea with increased work of breathing
R. Admit to telemetry unit, add IV corticosteroids and mini-nebulizer treatments

A. The patient states they were terrified when they were fighting to breathe and the wheezing would not stop.
B. The nurse determines the patient’s strengths include adherence to their medication regimen.
C. The care plan includes the health problem of impaired gas exchange, etiology, bronchospasm.
D. At 10:00 AM, no wheezing on auscultation, pulse oximetry is 94%, the patient reports no anxiety; the outcome has been met.

A

D

RATIONALE:
d. The two-part evaluative statement includes a decision about how well the outcome was met, along with patient data or behaviors that support this decision. Outcomes may have been met, partially met, or not met. The nurse collects evaluative data to measure outcome achievement. While this may justify terminating the care plan, that is not necessarily so. Data to assess health problems and patient variables are collected during the first step of the nursing process.

89
Q

A nurse is writing nursing outcomes in the affective domain for a patient who is trying to stop smoking. Which outcome statement will the nurse include in the care plan?

A. “The patient will state the relationship between smoking and coronary artery disease.”
B. “After the teaching session, the patient will redemonstrate the proper application of a nicotine patch.”
C. “The patient will state they value a healthy body sufficiently to stop smoking prior to discharge.”
D. “The patient will state that any changes in cough should be reported to the health care provider”

A

C

RATIONALE:
c. Affective outcomes pertain to changes in patient values, beliefs, and attitudes. Cognitive outcomes involve increases in patient knowledge; psychomotor outcomes describe the patient’s achievement of new skills; physical changes are actual bodily changes in the patient (e.g., weight loss, increased muscle tone).

90
Q

A nurse writes the outcome for a patient who is trying to lose weight: “The patient will explain the relationship between weight loss, increased exercise, and decreased calorie intake.” This outcome reflects which domain of learning?

A. Cognitive
B. Psychomotor
C. Affective
D. Physical changes

A

A

RATIONALE:
a. Cognitive outcomes involve increases in patient knowledge; psychomotor outcomes describe the patient’s achievement of new skills; affective outcomes pertain to changes in patient values, beliefs, and attitudes; and physical changes are actual bodily changes in the patient (e.g., weight loss, increased muscle tone).

91
Q

A nurse is writing an evaluative statement for a patient who is trying to lower cholesterol through diet and exercise. Which evaluative statement is most correctly written?

A. “Outcome met.”
B. “1/21/25—Patient reports no change in diet.”
C. “Outcome not met. Patient reports no change in diet or activity level.”
D. “1/21/25—Outcome met. Cholesterol level has decreased 10 mg.”

A

D

RATIONALE:
d. The evaluative statement should contain a date; the words “outcome met,” “outcome partially met,” or “outcome not met”; and the patient data or behaviors that support this decision. The other answer choices are incomplete statements.

92
Q

A nurse is attempting to improve care on the pediatric unit of a hospital. Which improvements might the nurse employ when following the recommendations of the Institute of Medicine’s Committee on Quality of Health Care in America? Select all that apply.

A. Basing patient care on continuous healing relationships
B. Customizing care to reflect the competencies of the staff
C. Using evidence-based decision making
D. Having a charge nurse as the source of control
E. Using safety as a system priority
F. Recognizing the need for secrecy to protect patient privacy

A

A,C,E

RATIONALE:
a, c, e. Care should be based on continuous healing relationships and evidence-based decision making. Customization should be based on patient needs and values with the patient as the source of control. Safety should be used as a system priority, and the need for confidentiality of care with transparency for the patient and designated individuals, rather than secrecy is used.

93
Q

As part of a hospital-wide quality-assurance program, an electronic medical record review for the last 6 months reveals a higher incidence of falls on a specific unit. The nurse authoring the study refers to the review as what type of evaluation?

A. Quality by inspection
B. Retrospective evaluation
C. Concurrent study
D. Quality by indicator

A

B

RATIONALE:
b. Quality by inspection focuses on finding deficient workers and removing them. Concurrent evaluation uses direct observation of nursing care, patient interviews, and chart review to determine whether the specified evaluative criteria are met. Retrospective evaluation may use post discharge questionnaires, patient interviews (by telephone or face to face), or chart review (nursing audit) to collect data. Quality as opportunity focuses on finding opportunities for improvement and fosters an environment thriving on teamwork, with people sharing the skills and lessons they have learned.

94
Q

The nurse manager of a unit with an excellent safety record meets with staff to present the findings of a recent audit. The manager states, “We’re doing well, but I believe we can do better. Who’s got an idea to foster increased patient well-being and satisfaction?” This leader has demonstrated they value which process?

A. Quality assurance
B. Quality improvement
C. Process evaluation
D. Outcome evaluation

A

B

RATIONALE:
b. Unlike quality assurance, quality improvement (QI) is internally driven. QI focuses on patient care rather than organizational structure and processes rather than people, and has no end points. Its goal is improving quality rather than assuring quality. Process evaluation and outcome evaluation are types of quality-assurance programs.

95
Q

Nurses note that allowing patients to choose the time of their breakfast to improve patient satisfaction has resulted in medication delays for patients who have prescriptions for medications taken on an empty stomach. Which action will direct the nurses to the best outcome?

A. Asking the pharmacy to dispense the medication at bedtime
B. Suggesting a quality improvement project piloting a 6:00 AM administration
C. Requesting that the health care provider prescribe the medication for midnight
D. Telling the nurse manager that patients are getting their medications late

A

B

RATIONALE:
b. Quality improvement or continuous quality improvement involves systematic, continuous actions that lead to measurable improvement in health care services and the health status of targeted patient groups. Quality-assurance programs enable nursing to be accountable for the quality of nursing care. Making changes without gathering needed data may prove unsafe or a waste of time. Reporting to the nurse manager does not reflect professional commitment to improving processes.

96
Q

The nurse on a medical-surgical unit attends a class on the seven crucial conversations in health care. After observing a colleague administer an incorrect dose of medication without reporting it, which action will the nurse take?

A. Speak to the nurse privately and tell her if she does not complete an event report, you will report her to the unit manager
B. Tell the nurse you overheard her discussing giving too much medication, and she must complete an event report or you will
C. Explain that you are aware of the medication incident, and you can assist her in notifying the health care provider for patient safety
D. Give the nurse a copy of the handout from the class and explain that this class in crucial conversations was very helpful

A

C

RATIONALE:
c. Crucial Conversations for Healthcare, addresses “undiscussable” communication breakdowns and gaps that can result in patient harm, medical errors, and staff turnover. When nurses feel unsafe to report problems or are not heard, dangerous shortcuts, incompetence, and disrespect may ensue. Threatening the nurse with reporting them is unprofessional and inconsistent with the crucial conversations. Providing a handout is passive aggressive and does not clearly relate the medication error to this concept.

97
Q

A nurse is using the classic elements of evaluation when caring for patients. Place the steps of evaluation in the proper order they are carried out.

A. Interpreting and summarizing findings
B. Collecting data to determine whether evaluative criteria and standards are met
C. Documenting your judgment
D. Terminating, continuing, or modifying the plan
E. Identifying evaluative criteria and standards (i.e., expected patient outcomes)

A

E,B,A,C,D

RATIONALE:
e, b, a, c, d. The five classic elements of evaluation in order are (1) identifying evaluative criteria and standards (what the nurse looks for during evaluation, that is, expected patient outcomes); (2) collecting data to determine whether these criteria and standards are met; (3) interpreting and summarizing findings; (4) documenting your judgment; and (5) terminating, continuing, or modifying the plan.

98
Q

The care plan for a patient just diagnosed with diabetes contains the expected outcome: “the patient will correctly measure the insulin dose and self-administer the injection, using correct technique by 12/12/24.” The nurse observes the client fumbled with the syringe and drew up less insulin than prescribed. What action will the nurse take first?

A. Document that the plan of care was unsuccessful
B. State continuation of the care plan is indicated
C. Assess the patient’s vision and dexterity and revise the plan
D. Designate a family member to administer the insulin

A

C

RATIONALE:
c. When an outcome is not achieved, the nurse can (1) delete or modify a diagnosis; (2) revise the diagnosis, making it more realistic; (3) adjust the time criteria, or (4) modify the nursing interventions. This outcome was not successfully met; further assessment and revision is indicated. It is inappropriate for the nurse to designate a family member to take over insulin administration without additional assessment and patient permission.

99
Q

A patient recovering from a myocardial infarction tells the nurse: “My life feels out of control since I had the heart attack. I’d like to try yoga, but I don’t think I’m strong enough to hold poses for long.” What response by the nurse is most appropriate?

A. “Concentrate on relaxing and taking your blood pressure medicine regularly; don’t worry about doing yoga.”
B. “Yoga can lower stress. Seek a slow-paced, gentle class when you are cleared for activity.”
C. “Ashtanga yoga is a gently paced yoga that can help with breathing and blood pressure.”
D. “Yoga is contraindicated for patients who have had a heart attack.”

A

B

RATIONALE:
b. Kripalu, or “gentle yoga,” focuses on relaxation and coming into balance. Ashtanga focuses on synchronizing breath with a fast-paced series of postures. The nurse should not discourage the use of yoga in patients who are healthy enough to participate. Yoga is not contraindicated in patients with controlled high blood pressure. Post-MI patients should resume activity with guidance from the health care provider.

100
Q

A nurse is providing education on CHAs to a group of patients in a rehabilitation facility. Which statement is appropriate for the nurse to include?

A. “CHAs are safe interventions used to supplement traditional care.”
B. “Many patients use CHAs as outpatients but do not wish to continue as inpatients.”
C. “Many nurses incorporate CHAs; could we try a relaxation exercise together?”
D. “Most CHAs are newer, and effectiveness has not been established.

A

C

RATIONALE:
c. Many nurses are expanding their clinical practice by incorporating CHAs. Although many CHAs seem safe, some therapies have led to harmful or lethal outcomes. Patients should seek guidance from the provider. Many patients use these types of therapies as outpatients and choose to continue them as inpatients. Most complementary and alternative therapies predate modern medicine; it was not until recently that nursing and medical schools began to teach about their use.

101
Q

A nurse mentor is teaching a new nurse about the underlying beliefs of CHAs versus allopathic therapies. Which statements by the new nurse indicate that teaching was effective? Select all that apply.

A. “CHA proponents believe the mind, body, and spirit are integrated and together influence health and illness.”
B. “CHA proponents believe that health is a balance of body systems: mental, social, and spiritual as well as physical.”
C. “Allopathy proponents believe that the main cause of illness is an imbalance or disharmony in the body systems.”
D. “Curing, according to CHA proponents, seeks to destroy the invading organism or repair the affected part.”
E. “Allopathic proponents place emphasis on disease, and drugs, surgery, and radiation are key tools for curing.”
F. “According to CHA proponents, health is the absence of disease.”

A

A,B,E

RATIONALE:
a, b, e. With CHAs, mind, body, and spirit are integrated and together influence health and illness; illness is a manifestation of imbalance or disharmony. Allopathic beliefs include: The main causes of illness are considered to be pathogens (bacteria or viruses) or biochemical imbalances, curing seeks to destroy the invading organism or repair the affected part, and emphasis is on disease and high technology. Drugs, surgery, and radiation are among the key tools for dealing with medical problems. According to allopathic beliefs, health is the absence of disease.

102
Q

A nurse plans to include integrative care for a patient who has many functional limitations related to painful rheumatoid arthritis. Which nursing intervention best represents the use of integrative care?

A. The nurse administers naproxen and uses guided imagery to promote relaxation and promote pain relief.
B. The nurse prepares the patient’s health care provider–approved herbal tea and uses meditation to relax the patient prior to bed.
C. The nurse administers naproxen and performs prescribed range-of-motion exercises.
D. The nurse arranges for acupuncture for the patient and designs a menu high in omega-3 fatty acids.

A

A

RATIONALE:
a. Patient participation in guided imagery (a CHA) with the administration of pain medications (allopathy) is an example of integrative care. Integrative care uses some combination of allopathic medicine and CHAs.

103
Q

A nurse works in a practice with naturopathic health care providers. What nursing actions would this nurse perform? Select all that apply.

A. Treating the symptoms of the disease
B. Providing patient education
C. Focusing on treating individual body systems
D. Suggesting interventions to prevent illness
E. Incorporating the healing power of nature
F. Encouraging patients’ responsibility for their own health

A

B,D,E,F

RATIONALE:
b, d, e, f. Naturopathic medicine is a system of medicine and a way of life. Emphasis is on patient responsibility, patient education, health maintenance, and disease prevention. Its principles include minimizing harmful side effects and avoiding suppression of symptoms, educating patients and encouraging them to take responsibility for their own health, treating the whole person, preventing illness, believing in the healing power of nature, and treating the cause of a disease or condition rather than its symptoms.

104
Q

A nurse cares for patients in a chiropractic office. What patient education would the nurse perform? Select all that apply.

A. Proper application of heat or ice
B. Explaining the purpose of electrical stimulation
C. Teaching relaxation techniques
D. Educating about a prescription medication
E. Teaching before an invasive procedure
F. Using safety with dietary supplements

A

A,B,C,F

RATIONALE:
a, b, c, f. Chiropractors may combine the use of spinal adjustments and other manual therapies with several approaches including heat and ice, electrical stimulation, relaxation techniques, rehabilitative and general exercise, counseling about weight and diet, and using dietary supplements. Chiropractors do not prescribe medication or perform invasive procedures.

105
Q

A nurse colleague with severe arthritis of the hip tells you they would like to incorporate use of medical marijuana into their pain relief regimen. What is most appropriate to share with your colleague?

A. Marijuana is a controlled substance.
B. Even where marijuana is legal, employees testing positive for THC may be disciplined.
C. This is an effective tool for many people for pain management.
D. The regulation of marijuana for recreation use is different than for employment.

A

B

RATIONALE:
b. THC is the psychoactive substance found in marijuana. Marijuana used for health problems or recreational use may be legal based on state law. However, for persons employed in safety-sensitive positions, such as nursing, the presence of THC or its constituents may pose a threat to health and safety. A positive test for THC may result in disciplinary action by an employer.

106
Q

A nurse is guiding a group of patients in a meditation practice. Which teaching point is most appropriate to help the patients achieve a state of calmness, physical relaxation, and psychological balance?

A. Always lie down in a comfortable position during meditation.
B. Focus on multiple problems that they feel demands attention.
C. Allow distractions to come and go naturally without judging them.
D. Suppress distracting or wandering thoughts to maintain focus.

A

C

RATIONALE:
c. Meditators are encouraged to let distractions come and go naturally without judging them. Assuming a comfortable posture lying down, sitting, standing, or walking is appropriate, as is focusing attention on a mantra, object, or breathing. The individual should not suppress distracting or wandering thoughts; instead, they should gently bring attention back to focus.

107
Q

A nurse working in an inpatient hospice facility incorporates aromatherapy into their practice. For which patient problem will ginger be most helpful?

A. Insomnia
B. Nausea
C. Dementia
D. Pain

A

B

RATIONALE:
b. Commonly used essential oils in a health care setting are ginger or peppermint for nausea and lavender or chamomile for insomnia.

108
Q

A nurse manager working in a hospital setting is researching the use of energy healing as an integrative care practice. Which patient would most likely benefit from this type of CHA?

A. One who is anxious about residual pain from cervical spinal surgery
B. One who is experiencing abdominal discomfort
C. One who is experiencing chronic pain from diabetes
D. One who has frequent cluster headaches

A

A

RATIONALE:
a. Energy healing is focused on pain that lingers after an injury heals as well as pain complicated by trauma, anxiety, or depression. Nutritional and herbal remedies treat all chronic pain, but especially abdominal discomfort, headaches, and inflammatory conditions, such as rheumatoid arthritis.

109
Q

A nurse in the pain management clinic is explaining complementary health approaches to a patient with severe back pain who is interested in holistic health. What approaches could the nurse include in the discussion? Select all that apply.

A. Acupressure or acupuncture
B. Qi gong or tai chi
C. Minimally invasive surgery
D. Gentle yoga
E. Aromatherapy

A

A,B,D,E

RATIONALE:
a, b, d, e. Holistic nursing is based on the philosophy of holism, which incorporates the mind, body, and emotions. Further, the interrelationships of the bio-psycho-social-spiritual dimensions of the person are greater than and not separate from their parts. Conventional scientific models focus on treatment of health conditions, individual organs, or organ systems rather than the whole dynamic being as part of their environment. Surgical procedures do not address body, mind, and emotions.

110
Q

A patient with chronic pain tells the nurse they are very tired and sleeping fitfully. What actions promoting sleep hygiene will the nurse suggest?

A. Sleep with a night light on.
B. Play soft, pleasing music.
C. Perform a relaxation exercise.
D. Establish a regular sleep–wake schedule.
E. Minimize noise.

A

B,C,D,E,F

RATIONALE:
b, c, d, e, f. Creating an optimal sleep environment for deep, restorative rest includes establishing a regular sleep–wake schedule and minimizing light and noise. Music can promote relaxation and increase sleep quality, as can relaxation exercises.

111
Q

Thirty-six hours after having surgery, a patient has a slightly elevated body temperature and generalized malaise as well as pain with redness at the surgical site. Which action is most appropriate?

MAR
Acetaminophen 650 mg every 6 hours prn fever
Cefazolin (antibiotic) 1 g 1 hour preoperatively
Cefazolin 1 g, every 6 hours 3 times, postoperatively

A. Documenting the findings and continuing to monitor the patient
B. Administering antipyretics and contacting the provider for an antibiotic prescription
C. Increasing the frequency of assessment to every hour and notifying the patient’s primary care provider
D. Obtaining a wound culture and increasing the frequency of wound care

A

A

RATIONALE:
a. The assessment findings are normal for this stage of healing following surgery. The patient is in the inflammatory phase of the healing process, which involves a response by the immune system. This acute inflammation is characterized by pain, heat, redness, and swelling at the site of the injury (surgery, in this case). The patient also has a generalized body response, including a mildly elevated temperature, leukocytosis, and generalized malaise.

112
Q

A nurse on a surgical unit has assessed and documented a patient’s wound and drainage. Which statements most accurately describe the characteristic of the wound drainage?

Graphic Record
T 99.9 P100 RR 20 BP 138/88
Nursing note: Patient postoperative day 2. Dry sterile dressing changed on abdominal incision. Incision edges are well approximated with a slight ½-cm opening at inferior edge; incisional edges reddened. Hemovac draining sanguineous material, 60 mL for the shift. Patient reports moderate pain, relieved by oxycodone X1.

A. Sanguineous drainage is composed of the clear portion of the blood and serous membranes.
B. Sanguineous drainage is composed of a large number of red blood cells and looks like blood.
C. Sanguineous drainage is composed of white blood cells, dead tissue, and bacteria.
D. Sanguineous drainage is thin, cloudy, and watery and may have a musty or foul odor.

A

B

RATIONALE:
b. Sanguineous drainage consists of large numbers of red blood cells and looks like blood. Bright-red sanguineous drainage is indicative of fresh bleeding, whereas darker drainage indicates older bleeding. Serous drainage, generally watery, is composed primarily of the clear, serous portion of the blood and serous membranes. Purulent drainage is made up of white blood cells, liquefied dead tissue debris, and both dead and live bacteria. It is thick, often has a musty or foul odor, and varies in color (such as dark yellow or green), depending on the causative organism.

113
Q

A postoperative patient who has a large abdominal incision suddenly calls out for help, shouting, “Something is falling out of my incision!” The nurse notes the wound is gaping open with tissue bulging outward. Place the nursing interventions in the order they should be performed, arranged from first to last.

A. Notify the health care provider of the situation.
B. Cover exposed tissue with sterile towels moistened with sterile 0.9% sodium chloride solution.
C. Place the patient in the low Fowler position.
D. Document the findings and outcome of interventions.
E. Maintain NPO status for return to the OR for repair.

A

C,B,A,E,D

RATIONALE:
c, b, a. e. d. The correct order of nursing interventions for this postoperative emergency is to place the patient in the low Fowler position (to prevent further damage or protrusion from increased intraabdominal pressure), cover exposed tissue with sterile towels moistened with sterile 0.9% sodium chloride solution (to protect the viscera), and notify the health care provider of the situation (to address the issue, likely with surgery). The patient is kept NPO, as prompt surgical repair will be needed. After the patient has received attention, the nurse documents all assessments and interventions in a timely manner.

114
Q

A patient was in an automobile accident and received a wound across the nose and cheek. After surgery to repair the wound, the patient says, “I am so ugly now.” Based on this statement, psychosocial problem will the nurse plan to address?

A. Pain
B. Wound healing
C. Body image
D. Change in cognition

A

C

RATIONALE:
c. Wounds cause emotional as well as physical stress.

115
Q

A patient is admitted with a nonhealing surgical wound. Which nursing interventions will the nurse use to promote wound healing? Select all that apply.

A. Applying sterile dressing supplies
B. Discussing zinc supplementation with the health care provider
C. Maintaining bedrest
D. Performing careful hand hygiene
E. Teaching the patient to increase protein in the diet
F. Suggesting the patient consume vitamin C–containing foods.

A

A,B,D,E,F

RATIONALE:
a, b, d, e, f. Careful hand washing (medical asepsis) is the most important. The nurse will use sterile dressings and supplies and promote intake of vitamins, zinc, and protein. Depending on the site of the wound and condition of the patient, bedrest may be indicated.

116
Q

A nurse on a surgical unit is working with a nursing student and discussing various phases of wound healing for postoperative patients. Which statements accurately describe these stages? Select all that apply.

A. Hemostasis occurs immediately after the initial injury.
B. A liquid called exudate is formed during the proliferation phase.
C. White blood cells move to the wound in the inflammatory phase.
D. Granulation tissue forms in the inflammatory phase.
E. During the inflammatory phase, the patient has generalized body response.
F. A scar forms during the proliferation phase.

A

A,C,E

RATIONALE:
a, c, e. Hemostasis occurs immediately after the initial injury, and exudate occurs in this phase as plasma and blood components leak out into the injured area. White blood cells, predominantly leukocytes and macrophages, move to the wound in the inflammatory phase to ingest bacteria and cellular debris. During the inflammatory phase, the patient has a generalized body response, including a mildly elevated temperature, leukocytosis (increased number of white blood cells in the blood), and generalized malaise. New tissue, called granulation tissue, forms the foundation for scar tissue development in the proliferation phase. New collagen continues to be deposited in the maturation phase, which forms a scar.

117
Q

The nurse preceptor is supervising a new graduate nurse as they assess a patient with a pressure injury. The graduate nurse documents the presence of biofilm in the wound. The preceptor recognizes the graduate nurse understands this concept when the graduate makes which of these statements? Select all that apply.

A. Enhanced healing occurs due to the presence of sugars and proteins.
B. Delayed healing develops due to dead tissue present in the wound.
C. Antibiotics against the bacteria become less effective.
D. Skin loses its integrity due to overhydration of the cells of the wound.
E. Delayed healing due to cells dehydrating and dying occurs.
F. Decreased effectiveness of the patient’s normal immune process results.

A

C,F

RATIONALE:
c, f. Wound biofilms are the result of wound bacteria growing in clumps, embedded in a thick, self-made, protective, slimy barrier of sugars and proteins. This barrier contributes to decreased effectiveness of antibiotics against the bacteria (antibiotic resistance) and decreases the effectiveness of the normal immune response by the patient (Baranoski & Ayello, 2020). Necrosis (dead tissue) in the wound delays healing. Maceration or overhydration of cells related to urinary and fecal incontinence can lead to impaired skin integrity. Desiccation is the process of drying up, in which cells dehydrate and die in a dry environment.

118
Q

The nurse is cleaning an open abdominal wound that has edges that are not approximated. What are accurate steps in this procedure? Select all that apply.

A. Use standard precautions or transmission-based precautions when indicated.
B. Moisten a sterile gauze pad or swab with the prescribed cleansing agent and squeeze out excess solution.
C. Clean the wound in full or half circles beginning on the outside and working toward the center.
D. Work outward from the incision in lines that are parallel to it from the dirty area to the clean area.
E. Clean to at least 1 inch beyond the end of the new dressing if one is being applied.
F. Clean to at least 3 inches beyond the wound if a new dressing is not being applied.

A

A,D,E

RATIONALE:
a, b, e. The correct procedure for cleaning an open wound with edges that are not approximated is: (1) use standard precautions and appropriate transmission-based precautions when indicated, (2) moisten sterile gauze pad or swab with prescribed cleansing agent and squeeze out excess solution, (3) use a new swab or gauze for each circle, (4) clean the wound in full or half circles beginning in the center and working toward the outside, (5) clean to at least 1 inch beyond the end of the new dressing, and (6) clean to at least 2 inches beyond the wound margins if a dressing is not being applied.

119
Q

A nurse is developing a care plan for an older adult patient who is recovering from a hip arthroplasty (hip replacement). Which assessment findings indicate a high risk for this patient to develop area(s) of pressure injury? Select all that apply.

A. The patient takes time to think about responses to questions.
B. The patient is an older adult with a poor appetite.
C. The patient reports inability to control their urine.
D. The patient’s albumin level is <3.2 mg/dL (normal, 3.4 to 5.4 g/dL).
E. Lab findings include BUN 12 (older adult, normal 8 to 23 mg/dL) and creatinine 0.9 (adult female, normal 0.61 to 1 mg/dL).
F. The patient reports increased pain in right hip when repositioning in bed or chair.

A

B,C,D,F

RATIONALE:
b, c, d, f. Pressure, friction, and shear, as well as other factors, usually combine to contribute to pressure injury development. The skin of older adults is more susceptible to injury; incontinence contributes to prolonged moisture on the skin, as well as negative effects related to urine in contact with skin; hip surgery involves decreased mobility during the postoperative period, as well as pain with movement, contributing to immobility; and increased pain in the hip may contribute to increased immobility. A low albumin level signals a risk for poor wound healing related to malnutrition. Apathy, confusion, and/or altered mental status are risk factors for pressure injury development; however, taking time to formulate responses is consistent with normal aging. This patient’s BUN and creatinine are within normal range; however, dehydration (indicated by an elevated BUN and creatinine) is a risk for pressure injury development.

120
Q

A nurse is explaining to a patient the anticipated effect of the application of cold to an injured area. What response indicates that the patient understands the explanation?

A. “There will be more discomfort in the area where the cold is applied.”
B. “I should expect more drainage from the incision after the ice has been in place.”
C. “Redness and swelling should decrease after cold treatment.”
D. “My incision may bleed more when the ice is first applied.”

A

C

RATIONALE:
c. The local application of cold constricts peripheral blood vessels, reduces muscle spasms, and promotes comfort. Cold reduces blood flow to tissues, decreases the local release of pain-producing substances, decreases metabolic needs, and capillary permeability. The resulting effects include decreased edema, coagulation of blood at the wound site, promotion of comfort, decreased drainage from wound, and decreased bleeding.

121
Q

A nurse is providing education to a patient and their family regarding the use of negative pressure wound therapy (NPWT). The nurse documents that the teaching has been effective when the patient and family make which statement?

A. “This therapy is used to collect excess blood loss and prevent formation of a scab.”
B. “The suction created will prevent infection and promote wound healing with less scar tissue.”
C. “Suction stimulates blood flow to the wound, removes excess fluid, and promotes a moist environment for healing.”
D. “This treatment irrigates the wound, suctions the irrigation fluid from the wound, and keeps it free from debris wound exudate.”

A

C

RATIONALE:
c. Negative pressure wound therapy (NPWT) promotes wound healing and wound closure through the application of uniform negative pressure on the wound bed, reduction in bacteria in the wound, and the removal of excess wound fluid, while providing a moist wound healing environment. The negative pressure results in mechanical tension on the wound tissues, stimulating cell proliferation, blood flow to wounds, and the growth of new blood vessels. It is used to treat a variety of acute or chronic wounds, wounds with heavy drainage, wounds failing to heal, or healing slowly.

122
Q

After an initial skin assessment, the nurse documents the presence pressure area that is reddened and has a 1-cm blister. How will the nurse document the wound stage?

A. Stage 1 dark maroon wound, skin intact
B. Stage 2 with 1-cm blister noted
C. Stage 3 wound base with red granulation tissue
D. Stage 4 blanchable reddened area, 2 cm

A

B

RATIONALE:
b. A stage 2 pressure injury involves partial-thickness loss of dermis and presents as a shallow open ulcer with a red–pink wound bed, without slough. It may also present as an intact or open/ruptured serum-filled blister. Dark maroon or purple wounds with intact skin represent deep tissue injury. Red granulation tissue is present in stage 3 or 4 pressure injuries that are healing. A blanchable, red area is a stage 1 pressure injury.

123
Q

A nurse notes a pressure wound base is red. Using the RYB system for documentation, what intervention is indicated?

A. Irrigating the wound and applying an absorbent dressing
B. Gently cleansing the wound and applying a moist dressing
C. Discussing consultation for surgical debridement with the provider
D. Performing frequent dressing changes to keep the wound and dressing dry

A

B

RATIONALE:
b. Red wounds are in the proliferative stage of healing and reflect the color of normal granulation tissue. Wounds in this stage need protection with nursing interventions that include gentle cleansing, use of moist dressings, and dressing changes only when necessary (or based on product manufacturer’s recommendations). To cleanse yellow wounds, nursing interventions include the use of wound cleansers and irrigation. The eschar found in black wounds requires debridement (removal) before the wound can heal.

124
Q

A nurse is developing education for nurses and UAPs related to prevention of pressure injuries for residents in a long-term care facility. Which action to prevent pressure injury will the nurses delegate to the UAP?

A. Maintaining the head of the bed elevated consistently
B. Massaging over bony prominences
C. Repositioning bedbound patients every 4 hours
D. Using a mild cleansing agent when cleansing the skin

A

D

RATIONALE:
d. To prevent pressure injuries, the nurse teaches the UAP to cleanse the skin routinely and whenever soiling occurs by using a mild cleansing agent with minimal friction, and avoiding hot water. The nurse educates the UAP to minimize the effects of shearing force by limiting the amount of time the head of the bed is elevated, when possible. Bony prominences should not be massaged, and bed-bound patients should be repositioned every 2 hours.

125
Q

A nurse caring for a patient with a stage 3 pressure wound with tunneling. How will the nurse best assess the tunneled area?

A. Moisten a sterile, flexible applicator with saline and insert it gently into the wound at a 90-degree angle with the tip down.
B. Photograph the wound per policy and describe the estimated depth in centimeters.
C. Gently insert a sterile applicator into the wound and move it in a clockwise direction.
D. Insert a calibrated probe gently into the wound and mark the point that is even with the surrounding skin surface.

A

A

RATIONALE:
a. To measure the depth of a wound, the nurse should perform hand hygiene and apply gloves; moisten a sterile, flexible applicator with saline and insert it gently into the wound at a 90-degree angle with the tip down; mark the point on the swab that is even with the surrounding skin surface, or grasp the applicator with the thumb and forefinger at the point corresponding to the wound’s margin; and remove the swab and measure the depth with a ruler.

126
Q

Determine the patient’s risk for pressure injury using the Braden scale found in Figure33-7, based on information in the electronic health record (EHR).

EHR 1430 Admission Assessment
S: Patient admitted from nursing home for sepsis, confusion, ambulatory dysfunction.
B: 87-year-old patient, with history of heart failure and hypertension; comes to ED with shortness of breath and yellow sputum.
A: Lungs with crackles, pale, short of breath on exertion, pulse oximetry 88%, skin fragile.
Bedrest maintained. States has not eaten nor drank fluids for last 36 hours; incontinent of small amount of urine × 2. Responding to painful stimuli, not participating in turning or care.
R: Need orders for oxygen, sputum culture, activity level. Consider IV fluids. J. Smith RN.

A. No risk
B. Moderate risk
C. High risk
D. Very high risk

A

D

RATIONALE:
d. The patient is at very high risk for pressure injury. This patient responds only to painful stimulate (1); is occasionally moist (3); is bedridden (1); has not eaten (1), and requires maximum assistance for moving (1) for a total of 7 points. The Braden scale scoring is: a score of 19 to 23 indicates no risk; 15 to 18, mild risk; 13 to 14, moderate risk; 10 to 12, high risk; and 9 or lower, very high risk (Braden & Maklebust, 2005). In addition, nurses use clinical judgment to incorporate risk factors and/or other health problems into preventative interventions.

127
Q

The nurse is caring for a patient 1 day postoperative abdominal surgery. The nurse identifies the patient is at risk for wound dehiscence. What patient risk factor is consistent with development of this problem?

A. Cigar smoker
B. Wound drainage 120 mL over 24 hours
C. Height, 5′ 6″ and weight 240 lb
D. WBC count 9,500 c/mm3

A

A

RATIONALE:
a. Wound dehiscence is the partial or total separation of wound layers as a result of excessive stress on unhealed wounds. Patients at greater risk include obese or malnourished individuals; tobacco smokers; and those taking anticoagulants, who have infected wounds, or who experience excessive coughing, vomiting, or straining (Hinkle & Cheever, 2018). An increase in the flow of (serosanguineous) fluid from the wound between postoperative days 4 and 5 may be a sign of an impending dehiscence. The patient may say that “something has suddenly given way.”

128
Q

A nurse at a health fair calculates the body mass index (BMI) of a person who weighs 68 kg and is 165 cm (1.65 m) tall. How will the nurse document the BMI?

A. 25 kg/m2
B. 46 kg/m2
C. 68 kg/m2
D. 165 kg/m2

A

A.

RATIONALE:
a. Example: Weight = 68 kg, height = 165 cm (1.65 m), Calculation: 68 ÷ (1.65)2 = 24.98

129
Q

After administering an enteral feeding, a nurse evaluates the patient’s tolerance of the feeding. Which findings suggesting intolerance require collaboration with the dietician and health care provider? Select all that apply.

A. Nausea and/or vomiting
B. Weight gain
C. Bowel sounds 20/min
D. 200-mL gastric residual
E. Absence of diarrhea and constipation
F. Slight abdominal pain and distention

A

A,C,E

RATIONALE:
a, c, e. Criteria to consider when evaluating patient feeding tolerance include absence of nausea, vomiting, minimal or no gastric residual, absence of diarrhea and constipation, absence of abdominal pain and distention, presence of bowel sounds within normal limits.

130
Q

A nurse is feeding an older adult patient with dementia. What intervention will best promote nutritional intake?

A. Stroke the underside of the patient’s chin to promote swallowing.
B. Serve meals in different places and at different times.
C. Offer a whole tray of various foods to choose from.
D. Avoid between-meal snacks to ensure hunger at mealtime.

A

A

RATIONALE:
a. To feed a patient with dementia, the nurse should stroke the underside of the patient’s chin to promote swallowing, serve meals in the same place and at the same time, provide one food item at a time since a whole tray may be overwhelming, and provide between-meal snacks that are easy to consume using the hands.

131
Q

A patient with COPD is experiencing anorexia and weight loss. Which intervention would be most helpful in stimulating appetite in this patient?

A. Administering pain medication after meals.
B. Encouraging the patient’s family to bring food from home when possible.
C. Scheduling respiratory therapy nebulizer treatments before each meal.
D. Reinforcing the importance of eating what is delivered to them.

A

B

RATIONALE:
b. Food from home that the patient enjoys may stimulate them to eat. Pain medication should be given before meals, respiratory therapy should be scheduled after meals, and telling the patient what they must eat is no guarantee that they will comply.

132
Q

A nurse is feeding a patient who is experiencing dysphagia. Which nursing intervention will best prevent aspiration?

A. Feed the patient solids first and liquids last.
B. Place the bed in the semi-Fowler position during feeding.
C. Provide a 30-minute rest period prior to mealtime.
D. Provide a straw for the patient’s beverages and soups.

A

C

RATIONALE:
c.The nurse should provide a 30-minute rest period prior to mealtime to promote better swallowing. The nurse alternates solids and liquids when feeding the patient; sits the patient upright or, if on bedrest, elevates the head of the bed at a 90-degree angle; and initiates a nutrition consult for diet modification and food size and/or consistency. Straws are avoided in patients with dysphagia. Assessing breath sounds will help detect aspiration but not prevent it.

133
Q

During interprofessional rounds, the charge nurse and health care provider evaluate patients to determine their need for parenteral nutrition (PN). Which patients will be identified as candidates for this type of nutritional support? Select all that apply.

A. Patient with irritable bowel syndrome and intractable diarrhea
B. Patient with celiac disease not absorbing nutrients from the GI tract
C. Patient who is underweight and needs short-term nutritional support
D. Patient who is comatose and needs long-term nutritional support
E. Patient who has anorexia and refuses to take foods via the oral route
F. Patient with burns who has not been able to eat adequately for 5 days

A

A,B,F

RATIONALE:
a, b, f. Assessment criteria used to determine the need for PN include an inability to achieve or maintain enteral access, motility disorders, intractable diarrhea, impaired absorption of nutrients from the GI tract, and when oral intake has been or is expected to be inadequate over a 7- to 14-day period. PN promotes tissue healing and is a good choice for a patient with burns who has an inadequate diet with an increased need for calories and nutrients. Oral intake is the best method of feeding; the second-best method is via the enteral route. For short-term use (less than 4 weeks), a nasogastric or nasointestinal route is usually selected. A gastrostomy (enteral feeding) is the preferred route to deliver enteral nutrition in the patient who is comatose because the gastroesophageal sphincter remains intact, making regurgitation and aspiration less likely than with NG tube feedings. Patients who refuse to take food should not be force fed nutrients against their will.

134
Q

A nurse is feeding a patient who reports feeling nauseated and unable to eat what is being offered. What would be the most appropriate initial action of the nurse in this situation?

A. Remove the tray from the room.
B. Administer an antiemetic and encourage the patient to take small amounts.
C. Explore why the patient does not want to eat the food.
D. Offer high-calorie snacks such as pudding and ice cream.

A

A

RATIONALE:
a. The first action of the nurse when a patient has nausea is to remove the tray, which may have noxious odor, from the room. The nurse may then offer small amounts of foods and liquids such as crackers or ginger ale. The nurse may also administer a prescribed antiemetic and try small amounts of food when it takes effect

135
Q

A nurse is receiving report on a patient with alcoholism who will be transferred to the medical-surgical unit. Due to long-term alcohol exposure, the nurse plans for administration of which nutrient?

A. B vitamins
B. Lipids
C. Fluids
D. C vitamins

A

A

RATIONALE:
a. The need for B vitamins is increased in alcoholism because these nutrients are used to metabolize alcohol, thus depleting their supply. Obesity, dehydration, and vitamin C deficiency may be present, but these are not directly related to the effect of alcohol on the GI tract.

136
Q

A nursing student is caring for a patient who had a gastrostomy tube placement 12 hours ago. Which action by the student is correct?

A. Using a cotton-tipped applicator dipped into sterile saline solution and gently cleaning around the insertion site
B. Washing the area surrounding the tube with a wet washcloth and with soap and water.
C. Adjusting the external disk every 3 hours to avoid crusting around the tube.
D. Taping a gauze dressing over the site after cleansing it.
E. Assessing the gastric residual every 4 hours.
F. Discontinuing feedings when gastric residual volume is 120 mL.

A

A

RATIONALE:
a. When caring for a new gastrostomy tube, the nurse would use a cotton-tipped applicator dipped in sterile saline to gently cleanse the area, removing any crust or drainage. The nurse would not use a washcloth with soap and water on a new gastrostomy tube but may use this method if the site is healed. Also, once the sutures are removed, the nurse should rotate the external bumper 90 degrees once a day. The nurse should leave the site open to air. If drainage is present, one thickness of precut gauze should be placed under the external bumper and changed as needed to keep the area dry

137
Q

A nurse is assessing a patient who has been NPO (nothing by mouth) prior to abdominal surgery. The patient is ordered a clear liquid diet for breakfast, to advance to a house diet as tolerated. Which assessments would indicate to the nurse that the patient’s diet should not be advanced?

A. The patient consumed 75% of the liquids on the breakfast tray.
B. The patient tells you they are hungry.
C. The patient’s abdomen is soft, nondistended, with bowel sounds.
D. The patient reports fullness and diarrhea after breakfast.

A

D

RATIONALE:
d. Tolerance to a diet can be assessed by the following: absence of nausea, vomiting, and diarrhea; absence of feelings of fullness; absence of abdominal pain and distention; feelings of hunger; and the ability to consume at least 50% to 75% of the food on the meal tray.

138
Q

A patient hospitalized for a stroke has a prescription for continuous tube feedings through a small-bore nasogastric tube. Following tube placement, which action by the nurse best confirms correct tube placement?

A. Auscultating the bowel sounds
B. Measuring the pH of gastric aspirate
C. Measuring the amount of residual in the stomach
D. Ensuring validation of tube placement by x-ray

A

D

RATIONALE:
d. Radiographic examination is the most accurate method to validate tube placement in the stomach. In addition, the length of the exposed tube is measured after insertion and documented. Tube length should be checked and compared with this initial measurement, in conjunction with the previous two methods for checking tube placement. Other methods that can be used are aspiration of gastric contents and measurement of the pH of the aspirate. Visual assessment of aspirated gastric contents is also suggested as a tool to check placement. The auscultatory method is considered inaccurate and unreliable. Measurement of residual volume amount does not confirm placement.

139
Q

A nurse specializing in care of older adults speaks to a group of nursing students about that population’s challenge with obtaining sufficient nutrition. Which points will the nurse include in the discussion? Select all that apply.

A. An increase in BMR and physical activity require additional calories.
B. Tooth loss and periodontal disease may make chewing more difficult.
C. Decreased peristalsis can result in constipation, requiring additional fiber and fluid.
D. Loss of taste between sweet and salty occurs with a preference for sweets.
E. Older adults express an increase thirst sensation.
F. Caloric needs decrease, and the need for nutrients increases, especially protein.

A

B,C,D,F

RATIONALE:
b, c, d, f. Due to age-related changes, caloric needs of the body decrease; however, the need for nutrients, including protein, calcium, vitamin B12, and vitamin D increase. If tooth loss and periodontal disease occur, adequate nutritional intake may become more difficult. An increase in fiber and fluid intake can relieve constipation related to decreased peristalsis. Older adults are also prone to dehydration related to loss of thirst sensation. Loss of taste between sweet and salty occurs with a preference for sweets. Dietary restrictions related to chronic illness, limited income, isolation, and age-related physiologic changes place persons in this age group at risk for malnutrition.

140
Q

A nurse in the intensive care unit is reviewing diagnostic studies to evaluate a patient’s nutritional status. What findings consistent with inadequate nutrition require follow-up by the nurse? Select all that apply.

A. Decreased hemoglobin
B. Low prealbumin level
C. Increased transferrin
D. Anemia
E. Elevated lymphocytes

A

A,B,D,E

RATIONALE:
a, b, d, e. Test results for hemoglobin (normal = 12 to 18 g/dL): decrease indicates anemia. Results for hematocrit (normal = 40% to 50%): decrease indicates anemia; increase indicates dehydration. Serum albumin and prealbumin tests for malnutrition and malabsorption will be decreased, as will transferrin and blood urea nitrogen. Elevated lymphocytes may indicate infection.

141
Q

A nurse plans to administer a bolus tube feeding for a patient but is unable to aspirate gastric contents due to a clogged tube. What action will the nurse take next?

A. Use warm water or air, applying gentle pressure to remove the clog.
B. Use the tube’s stylet to unclog the tubes.
C. Administer a cola beverage to remove the clog.
D. Replace the tube with a new one.

A

A

RATIONALE:
a. To remove a clog in a feeding tube, the nurse should try using warm water or air and gentle pressure to unclog it. The stylet should never be used to unclog a tube, and cola and meat tenderizers have not been shown effective in removing clogs. The nurse should first attempt to remove the clog, and, if unsuccessful, the tube should be replaced.

142
Q

A nurse performs presurgical assessments of patients in an ambulatory care center. Which patient assessment requires collaboration with the surgeon, as the procedure could need to be postponed?

A. 19-year-old patient who is a vegan
B. Older adult patient who takes daily nutritional drinks
C. 43-year-old patient who takes ginkgo biloba and an aspirin daily
D. Infant who is breastfeeding

A

C

RATIONALE:
c. Patients taking gingko biloba (an herbal), aspirin, and vitamin E (dietary supplement) may have increased risk for excessive bleeding and may requiring surgery to be postponed. Being a vegan should not affect surgery unless the patient has serious nutritional deficiencies. Drinking nutritional drinks and breastfeeding do not adversely affect the outcomes of surgery.

143
Q

A nurse is caring for a patient with ill-fitting dentures. What modification to their diet will the nurse suggest?

A. Clear liquid
B. Full liquid
C. Mechanically altered
D. Honeylike liquids

A

C

RATIONALE:
c. Mechanically altered diets provide adequate in calories and nutrients and contain chopped, ground, or soft foods. Liquid diets are generally used as transitional diets when eating resumes after acute illness, surgery, or parenteral nutrition. Clear-liquid diets are inadequate in calories, protein, and most nutrients; progression to more nutritious alternatives is recommended as soon as possible. Full-liquid diets include clear liquids plus milk and milk drinks, puddings, custards, plain frozen desserts, pasteurized eggs, cereal gruels, vegetable juices, and milk and egg substitutes in addition to clear liquids. A high-calorie, high-protein supplement is recommended if a full-liquid diet is used for more than 3 days.

144
Q

A new graduate nurse and their preceptor must collect several urine specimens for laboratory testing. Which techniques for urine collection by the graduate nurse are performed incorrectly, requiring the preceptor to intervene? Select all that apply.

A. Catheterizing a patient to collect a sterile urine sample for routine urinalysis
B. Collecting a clean-catch urine specimen in the morning and storing it at room temperature until an afternoon pick-up
C. Collecting a sterile urine specimen from the collection bag of a patient’s indwelling catheter
D. Collecting about 3 mL of urine from a patient’s indwelling catheter to send for a urine culture
E. Planning to collect a sterile specimen from a patient with a urinary diversion by catheterizing the stoma
F. Discarding the first urine of the day when performing a 24-hour urine specimen collection on a patient

A

D,E,F

RATIONALE:
d, e, f. A urine culture requires about 3 mL of urine, whereas routine urinalysis requires at least 10 mL of urine. The preferred method of collecting a sterile urine specimen from a urinary diversion is to catheterize the stoma. For a 24-hour urine specimen, the nurse should discard the first voiding, then collect all urine voided for the next 24 hours. A sterile urine specimen is not required for a routine urinalysis. Urine chemistry is altered after urine stands at room temperature for a long period of time. A specimen from the collection drainage bag may not be fresh urine and could result in an inaccurate analysis.

145
Q

A nurse caring for older adults in an extended-care facility performs regular assessments of the patients’ urinary functioning. Which patients would the nurse identify as at risk for urinary retention? Select all that apply.

A. Patient who is diagnosed with an enlarged prostate
B. Patient who is on bedrest
C. Patient who is diagnosed with vaginal prolapse
D. Older adult patient with dementia
E. Patient who is taking antihistamines to treat allergies
F. Patient who has difficulty walking to the bathroom

A

A,C,E

RATIONALE:
a, c, e. Urinary retention occurs when urine is produced normally but is not excreted completely from the bladder. Factors associated with urinary retention include medications such as antihistamines, an enlarged prostate, or vaginal prolapse. Being on bedrest, having dementia, and having difficulty walking to the bathroom may place patients at risk for urinary incontinence.

146
Q

A nurse in the gynecology clinic is preparing an educational brochure to teach patients how to prevent UTIs. Which teaching points would the nurse include? Select all that apply.

A. Wear underwear with a cotton crotch.
B. Take baths rather than showers.
C. Drink of six to eight 8-oz glasses of liquid per day.
D. Urinate before and after intercourse.
E. After defecation, dry the perineal area from the front to the back.
F. Observe the urine for color, amount, odor, and frequency.

A

A,C,E,F

RATIONALE:
a, c, e, f. It is recommended that a healthy adult drink six to eight 8-oz glasses of fluid daily, dry the perineal area after urination or defecation from the front to the back, and observe the urine for color, amount, odor, and frequency. It is also recommended to wear underwear with a cotton crotch, take showers rather than baths, and drink two glasses of water before and after sexual intercourse and void immediately after intercourse. Observing urine characteristics will not prevent a UTI; however, this observation may help a patient notice an infection.

147
Q

A patient who has pneumonia has had a fever for 3 days. What characteristics would the nurse anticipate related to the patient’s urine output?

A. Decreased amount and highly concentrated
B. Decreased amount and very pale like water
C. Increased amount and very concentrated
D. Increased amount and dilute appearing

A

A

RATIONALE:
a. Fever and diaphoresis cause the kidneys to conserve body fluids. Thus, the urine is concentrated and decreased in amount.

148
Q

The health care provider has ordered an indwelling catheter to be inserted to relieve urinary retention in a male patient with prostate enlargement. What consideration will the nurse keep in mind when performing this procedure?

A. The male urethra is more vulnerable to injury during insertion.
B. In the hospital, a clean technique is used for catheter insertion.
C. The catheter is inserted 2 to 3 inches into the meatus.
D. Since it uses a closed system, the risk for UTI is absent.

A

A

RATIONALE:
a. Because of the length of the male urethra and need to insert the catheter 6 to 8 inches, it is more prone to injury. The nurse inserts the catheter for a female patient 2 to 3 inches. This procedure requires surgical asepsis to prevent introducing bacteria into the urinary tract. The presence of an indwelling catheter places the patient at risk for a UTI.

149
Q

A nurse is caring for a patient with an enlarged prostate who has had an indwelling catheter for several weeks. A prescription for continuous bladder irrigation (CBI) is written after the patient developed hematuria post cystoscopy. The nurse teaches the patient the purpose of CBI is to prevent what situation?

A. Catheter infection due to long-term use
B. Need to flush the catheter of organisms post procedure
C. Blood clots that could block the catheter
D. Need for increased fluid intake

A

C

RATIONALE:
c. Post procedure continuous bladder irrigation, in the presence of hematuria, prevents stasis of blood and clot formation potentially obstructing urine output. In the absence of hematuria, clots or debris, natural irrigation of the catheter through increased fluid intake by the patient is preferred. It is preferable to avoid catheter irrigation unless necessary to relieve or prevent obstruction (Gould et al., 2009; SUNA, 2015).

150
Q

A nurse is caring for a patient who has a urinary diversion (urostomy) after cystectomy (removal of the bladder) to treat bladder cancer. What interventions are indicated for this patient? Select all that apply.

A. Measuring the patient’s fluid intake and output
B. Keeping the skin around the stoma moist
C. Emptying the appliance frequently
D. Reporting any mucus in the urine to the primary care provider
E. Encouraging the patient to look away when changing the appliance
F. Monitoring the return of intestinal function and peristalsis

A

A,C,F

RATIONALE:
a, c, f. Urinary diversion involves the surgical creation of an alternate route for excretion of urine. When caring for a patient with a urinary diversion, the nurse should measure the patient’s fluid intake and output to monitor fluid balance, change the appliance frequently, monitor the return of intestinal function and peristalsis, keep the skin around the stoma dry, watch for mucus in the urine as a normal finding, and encourage the patient to participate in care and look at the stoma.

151
Q

A nurse is changing the stoma appliance on a patient’s ileal conduit. Which finding requires the nurse to follow up with the provider?

A. Stoma is moist.
B. Skin around the stoma is irritated.
C. Urine is leaking from the stoma.
D. Stoma is a purple-black color.

A

D

RATIONALE:
d. The stoma should appear pink to red, shiny, and moist; a dark brown or purple-blue stoma may reflect compromised circulation. The nurse contacts the health care provider immediately. A urostomy is incontinent; urine leakage is expected.

152
Q

A postoperative patient is having difficulty voiding and reports suprapubic pressure. What action can the nurse take to promote voiding?

A. Pouring cold water over the patient’s fingers and perineum
B. Assessing bladder residual using the bladder scanner
C. Immediately encouraging the patient to void
D. Recommending an indwelling catheter

A

B

RATIONALE:
b. Factors associated with urinary retention include medications, an enlarged prostate, and vaginal prolapse. Assist the patient to void when the patient first feels the urge. Assessing for residual urine will not promote voiding; rather, it will determine the volume of urine in the bladder. Cold water would cause the patient to tighten their muscles.

153
Q

A nurse caring for a patient who just began hemodialysis assesses the patient’s AV fistula. Nursing documentation includes: “5/10/25 0930 AV fistula in the right forearm negative for thrill and bruit. Patient denies pain and tenderness.” Which finding is essential for the nurse report to the health care provider?

A. Thrill and bruit are absent.
B. Area is without redness or swelling.
C. Patient denies pain and tenderness.
D. Trace edema of the fingers is present.

A

A

RATIONALE:
a. The nurse palpates and auscultates over the access site, feeling for a thrill or vibration and listening for the bruit or swishing sound. Presence of the thrill and bruit are normal findings, indicating patency of the access. Decreased or absent thrill and/or bruit indicates a that there is an issue with the patency of the access, which could be a result of narrowing or clotting of the access, resulting in poor blood flow. No report of pain, redness, or swelling is a normal finding. A trace of edema is not a priority.

154
Q

A nurse is caring for an alert, ambulatory, older adult with urinary frequency who has difficulty making it to the bathroom in time. Which nursing intervention is most appropriate to include in the care plan for this patient?

A. Explaining that incontinence is an expected occurrence with aging
B. Asking the patient’s family/caregivers to purchase incontinence pads for the patient
C. Teaching the patient how to perform PFMT exercises at regular intervals
D. Inserting an indwelling catheter to prevent skin breakdown

A

C

RATIONALE:
c. Pelvic floor exercises (Kegel exercises) may help a patient regain control of the micturition. Incontinence is not a normal consequence of aging. Using absorbent products may remove motivation from the patient and caregiver to seek evaluation and treatment of the incontinence; they should be used only after careful evaluation by a health care provider. Due to risk for infection, an indwelling catheter is the last choice of treatment.

155
Q

A nurse is caring for a patient who is taking phenazopyridine (a urinary tract analgesic) for a UTI. The patient states, “My urine was bright orange-red today; I think I’m bleeding. Something is terribly wrong.” How will the nurse best respond?

A. “The medication causes a red-orange tinge to the urine; it is expected.”
B. “I will test your urine for blood.”
C. “This may be the result of an injury to your bladder.”
D. “I’ll hold the medication and let the provider know you are allergic to the drug.”

A

A

RATIONALE:
a. Phenazopyridine, a urinary tract analgesic, can cause orange or orange-red urine; the nurse educates the patient to expect this change.

156
Q

A nurse is caring for a male patient who had a urinary sheath applied following hip surgery. What nursing interventions are appropriate to include when caring for this patient? Select all that apply.

A. Preventing the tubing from kinking to maintain free urinary drainage
B. Changing the sheath weekly and provide hygiene
C. Fastening the sheath tightly to prevent the possibility of leakage
D. Having the patient maintain bedrest to prevent the sheath from slipping off
E. Leaving 1 to 2 inches (2.5 to 5 cm) beyond the tip of the penis
F. Ensuring the device does not restrict blood flow.

A

A,E,F

RATIONALE:
a, e, f. Maintaining free urinary drainage is a nursing priority. Institute measures to prevent the tubing from becoming kinked and urine from backing up in the tubing. The catheter should be allowed to drain freely through tubing that is not kinked. Nursing care of a patient with a urinary sheath includes skin care to prevent excoriation. Remove the condom daily and wash the penis with soap and water, and dry it carefully. Care must be taken to fasten the sheath securely enough to prevent leakage, yet not so tightly as to constrict the blood vessels in the area. The tip of the tubing should extend 1 to 2 inches (2.5 to 5 cm) beyond the tip of the penis to prevent irritation to the sensitive glans area. Confining a patient to bedrest increases the risk for hazards of immobility.

157
Q

A nurse receives a prescription to catheterize a patient following surgery. What nursing action reflects correct technique?

A. Planning to use different equipment for catheterization of male versus female patients
B. Selecting the smallest appropriate size indwelling urinary catheter
C. Sterilizing the equipment prior to insertion
D. Avoiding filling the balloon with sterile water to prevent pressure on tissues

A

B

RATIONALE:
b. The smallest appropriate indwelling urinary catheter should be selected to aid in prevention of CAUTIs in the adult hospitalized patient (ANA, 2014; SUNA, 2015). The equipment used for catheterization is usually prepackaged in a sterile, disposable tray and is the same for both male and female patients. Most kits already contain a standard-sized catheter. Catheters are graded on the French (F) scale according to lumen size, with 12 to 16Fr gauge commonly used (Bardsley, 2015a; Beauchemin et al., 2018; Newman, 2013). A 14F, 5-mL or 10-mL balloon is usually appropriate, unless ordered otherwise (ANA, 2014).

158
Q

A nurse in the emergency room is teaching a patient how to collect a midstream urine specimen. What instructions will the nurse give the patient? Select all that apply.

A. Wash your hands with soap and water.
B. Open the container and place the lid face down on the counter.
C. Separate your labia and wipe with the antiseptic towelettes in the kit.
D. Without letting go of the labia, void a small amount into the toilet or collection hat.
E. Lean the collection container against the urinary opening and void into the container.
F. Void an ounce, then remove the container and finish voiding in the toilet.

A

A,C,D,E

RATIONALE:
a, c, d, e. The nurse gives these instructions to collect the midstream/clean-catch urine specimen: Wash your hands with soap and water. Open the collection cup, and place the lid face up; do not touch the inside. Separate the labia and cleanse the urinary opening with soap and water or towelettes included in the kit. Void about 1 oz. (30 mL) into the toilet, then move the collection cup close to the urinary opening and void about 1 oz (no less than 2 teaspoons) into the container. Pass the remainder of the urine into the toilet. Without touching the inside of the lid, close the cup and return it to the nurse.

159
Q

A nurse on a pediatric surgical unit notes a 10-year-old child has developed nocturnal enuresis. What health concern will the nurse plan for?

A. Constipation
B. Bedwetting after the age of toilet training
C. Patient who is manipulative
D. Infection

A

B

RATIONALE:
b. Urinary incontinence of urine past the age of toilet training is termed enuresis. Hospitalization may cause regression of toileting habits.

160
Q

A nursing student hears in report that their patient is receiving a nephrotoxic medication. The student plans care to include what action?

A. Teaching the patient to expect increased voiding
B. Assessing for kidney damage
C. Preventing urinary incontinence
D. Observing for nocturia

A

B

RATIONALE:
b. Nephrotoxic medications are those capable of causing kidney damage. The nurse can assess I&O, quality of the urine, and renal function blood tests to detect this problem. Urinary frequency, incontinence, and getting up at night to void (nocturia) are not effects of nephrotoxic medications.

161
Q

A nurse is performing an abdominal assessment on a patient experiencing frequent bouts of diarrhea. The nurse first observes the contour of the abdomen, noting any masses, scars, or areas of distention. What action will the nurse perform next?

A. Auscultating the abdomen using an orderly clockwise approach in all abdominal quadrants
B. Percussing all quadrants of the abdomen in a systematic clockwise manner to identify masses, fluid, or air in the abdomen
C. Lightly palpating over the abdominal quadrants; first checking for any areas of pain or discomfort
D. Deeply palpating over the abdominal quadrants, noting muscular resistance, tenderness, organ enlargement, or masses

A

A

RATIONALE:
a. The sequence for abdominal assessment proceeds from inspection, auscultation, percussion, and then palpation. Inspection and auscultation are performed before palpation because palpation may disturb normal peristalsis and bowel motility.

162
Q

A nurse is administering a large-volume cleansing enema to a patient prior to surgery. When the enema solution is introduced, the patient reports severe cramping. What nursing intervention would the nurse perform next?

A. Elevating the head of the bed 30 degrees and repositioning the rectal tube
B. Placing the patient in a supine position and modifying the amount of solution
C. Lowering the solution container and checking the temperature and flow rate
D. Removing the rectal tube and notifying the primary care provider

A

C

RATIONALE:
c. If the patient reports severe cramping with introduction of an enema solution, the nurse should lower the solution container and check the temperature and flow rate. If the solution is too cold or the flow rate is too fast, severe cramping may occur. The head of the bed may be elevated 30 degrees for the patient’s comfort if the patient needs to be placed on a bedpan in the supine position while receiving the enema.

163
Q

A nurse working on a GI unit is caring for a group of patients. In patients with which health problems or issues could the assessment possibly reveal decreased or absent bowel sounds after listening for 2 minutes? Select all that apply.

A. Peritonitis
B. Prolonged bedrest
C. Diarrhea
D. Gastroenteritis
E. Early bowel obstruction
F. Postoperative paralytic ileus

A

A,B,F

RATIONALE:
a, b, f. Decreased or absent bowel sounds—documented only after listening for in the epigastric and umbilical area of the abdomen for 2 minutes or longer (Bickley et al., 2021)—signify the absence of bowel motility, commonly associated with peritonitis, paralytic ileus, and/or prolonged immobility. Hyperactive bowel sounds indicate increased bowel motility, commonly caused by diarrhea, gastroenteritis, or early bowel obstruction.

164
Q

A nurse in a long-term care facility is assessing a group of patients. In which patients would the nurse anticipate increased risk for developing diarrhea? Select all that apply.

A. Patient taking opioids for pain
B. Patient taking metformin for type 2 diabetes
C. Patient taking diuretics
D. Patient who developed dehydration
E. Patient taking amoxicillin clavulanate for infection
F. Patient taking magnesium-containing antacids

A

B,E,F

RATIONALE:
b, e, f. Diarrhea is a potential adverse effect of treatment with amoxicillin clavulanate, metformin, or antacids containing magnesium. Opioids, diuretics, and dehydration may lead to constipation.

165
Q

A nurse plans to administer a retention enema to a patient with a fecal impaction. Which nursing action is appropriate for this procedure?

A. Administering a large volume of solution (500 to 1,000 mL)
B. Mixing milk and molasses in equal parts for an enema
C. Instructing the patient to retain the enema for at least 30 minutes
D. Administering the enema while the patient is sitting on the toilet

A

C

RATIONALE:
c. Instruct the patient to retain the enema solution for at least 30 minutes or as indicated in the manufacturer’s instructions. The milk and molasses mixture is a carminative enema that helps to expel flatus. The patient should be instructed to lie on their left side as dictated by patient condition and comfort.

166
Q

A nurse prepares to assist a patient with a newly created ileostomy. Which recommended patient teaching points would the nurse stress? Select all that apply.

A. “When you inspect the stoma, it should be dark purple-blue.”
B. “The size of the stoma will stabilize within 2 weeks.”
C. “Keep the skin around the stoma site clean and moist.”
D. “The stool from an ileostomy is normally liquid.”
E. “Eat dark-green vegetables to control the odor of the stool.”
F. “You may have a tendency to develop food blockages.”

A

D,E,F

RATIONALE:
d, e, f. Ileostomies normally have liquid, foul-smelling stool. The nurse should encourage intake of dark-green vegetables for their chlorophyll content, which helps to deodorize the feces. Explain that patients with ileostomies may tend to develop food blockages, especially when high-fiber foods are consumed. The stoma should be dark pink to red and moist. Stoma size usually stabilizes within 4 to 6 weeks, and the skin around the stoma site (peristomal area) should be kept clean and dry.

167
Q

A nurse is preparing a hospitalized patient for a colonoscopy. Which nursing action is the recommended preparation for this test?

A. Having the patient consume a low-fiber diet several days before the test
B. Having the patient take bisacodyl and ingest a gallon oral polyethylene glycol solution (PEG)
C. Preparing the patient for the use of general anesthesia during the test
D. Explaining that barium contrast mixture will be given to drink before the test

A

A

RATIONALE:
a. If possible, a low-residue diet (low fiber) should be followed several days before the procedure, although some may have a full liquid diet the day before the procedure. There are multiple types of bowel preps for this procedure; the health care provider determines the best regimen for the individual.
The prep is usually given as a split dose, with half being given the night before and rest the morning of the procedure. It is recommended the second dose be given at least 5 hours before and completed at least 2 hours before the study. Occasionally, some patients may receive the prep the same day as the procedure, especially if it is scheduled for later in the day.
Conscious sedation, not general anesthesia, will be given for the colonoscopy. A chalky-tasting barium contrast mixture is given to drink before an upper GI and small-bowel series of tests.

168
Q

A nurse is performing digital removal of stool on a patient with a fecal impaction. During the procedure, the patient tells the nurse they are feeling dizzy and nauseated and then vomits. What should be the nurse’s next action?

A. Reassuring the patient that this is a normal reaction to the procedure
B. Stopping the procedure, preparing to administer CPR, and notifying the primary care provider
C. Stopping the procedure, assessing vital signs, and notifying the health care provider
D. Pausing the procedure, waiting 5 minutes, and then resuming the procedure

A

C

RATIONALE:
c. When a patient reports dizziness, lightheadedness, nausea, and/or vomiting during digital stool removal, the nurse recognizes the vagus nerve may have been stimulated. The nurse should stop the procedure, assess heart rate and blood pressure, and notify the health care provider.

169
Q

A nurse is scheduling tests for a patient who has been experiencing epigastric pain. The health care provider ordered the following tests: (a) barium enema, (b) fecal occult blood test, (c) endoscopic studies, and (d) an upper GI series. What is the correct order for performing the tests?

A. c, b, d, a
B. d, c, a, b
C. a, b, d, c
D. b, a, d, c

A

D

RATIONALE:
d. A fecal occult blood test is performed first to detect GI bleeding. Lower GI barium studies should be performed next to visualize GI structures and reveal any inflammation, ulcers, tumors, strictures, or other lesions. A barium enema and routine radiography should precede an upper GI series because retained barium from an upper GI series could take several days to pass through the GI tract and obscure anatomic detail on the barium enema studies. Noninvasive procedures usually take precedence over invasive procedures, such as endoscopic studies, when sufficient diagnostic data can be obtained from them.

170
Q

A nurse is caring for a patient who had abdominal surgery and has a nasogastric tube attached to low suction. Which nursing actions are appropriate when caring for this patient? Select all that apply.

A. Irrigating the tube with 30-mL normal saline solution
B. Confirming tube placement via pH testing of gastric secretions
C. Positioning the air vent at the level of the patient’s umbilicus
D. Instilling irrigation via the blue air vent
E. Monitoring the patient’s abdomen for distention
F. Documenting the nasogastric irrigation and drainage with I & O

A

A,B,D,E,F

RATIONALE:
a, b, d, e, f. Care of a patient with an NG tube connected to suction includes verifying placement before administration of any fluids or medications to avoid aspiration. Radiographic evidence of the tip of the tube in the stomach, measurement of tube length, measurement of tube marking, measurement of aspirate pH, and carbon dioxide monitoring may be used. The nurse irrigates the tube with 30 mL of saline solution (or as prescribed/ per policy) with an irrigating syringe. The nurse separates the tube from the suction device, clamps the NG tube to prevent gastric secretions from leaking, and places the tip of the syringe in the tube to gently insert the saline solution. The nurse should not place irrigant in the blue air vent of a Salem sump or double-lumen tube; rather, the nurse should instill air through the vent. The air vent decreases any pressure that has built up in the stomach during suction; instilling air promotes drainage. The air vent should be placed above the stomach. Abdominal distention may indicate lack of proper drainage or delayed return of GI function. The nurse documents all irrigation and NG tube output with the I & O. If the nurse allows the irrigant to drain out, that is excluded from intake.

171
Q

A nurse is planning a bowel program for a patient with frequent constipation after sustaining a spinal cord injury. What is the first step the nurse will take?

A. Offering a diet that is low in residue
B. Increasing fluid intake to 3,000 mL daily
C. Administering daily enemas to stimulate peristalsis
D. Assessing the patient’s bowel patterns

A

D

RATIONALE:
d. The nurse follows the steps of the nursing process to plan care. First the nurse assesses the patient’s bowel movements including frequency, consistency, shape, volume, and color. Based on the assessment findings, the nurse may recommend 3 of fluid daily or administer an enema, as appropriate. The nurse also monitors bowel sounds, teaches about specific foods that promote bowel regularity, ensures privacy, and encourages adequate fluid intake.

172
Q

A community health nurse is providing an adult education session about colon cancer. Which signs and symptoms of this cancer will the nurse include? Select all that apply.

A. Blood in the stool
B. Previous colonoscopy
C. Passing two large bowel movements daily
D. Unintentional weight loss
E. Upper abdominal cramping
F. Previous opioid use

A

A,D

RATIONALE:
a, d. The nurse discusses signs and symptoms of colon cancer including rectal bleeding or blood in the stool, persistent change in bowel pattern or consistency of stool, persistent cramping or pain in lower abdomen (or gas), a feeling of incomplete bowel emptying, weakness or fatigue, and unintentional weight loss.

173
Q

For a patient with which health problem or issue would a nurse expect the health care provider to order colostomy irrigation?

A. IBS
B. Left-sided end colostomy in the sigmoid colon
C. Postradiation damage to the bowel
D. Crohn disease

A

B

RATIONALE:
b. Irrigations are used to promote regular evacuation of distal colostomies. Colostomy irrigation may be indicated in patients who have a left-sided end colostomy in the descending or sigmoid colon, are mentally alert, have adequate vision, and have adequate manual dexterity needed to perform the procedure. Contraindications include IBS, peristomal hernia, postradiation damage to the bowel, diverticulitis, and Crohn disease (Kent et al., 2015).

174
Q

A nurse is assisting a patient to change an ostomy appliance when they note the stoma is protruding into the bag. What would be the nurse’s first action in this situation?

A. Reassuring the patient that this is a normal with a new ostomy
B. Notifying the health care provider that the stoma is prolapsed
C. Having the patient rest for 30 minutes to see if the prolapse resolves
D. Replacing the appliance with a larger appliance

A

C

RATIONALE:
c. If the stoma protrudes into the bag after changing the ostomy appliance, the nurse should have the patient rest for 30 minutes. If the stoma is not back to normal size within that time, notify the health care provider. If the stoma stays prolapsed, it may twist, resulting in impaired circulation to the stoma.

175
Q

A nurse is caring for an older adult who reports persistent constipation and has a number of laxative prescriptions on the MAR. Which medication would the nurse avoid for this patient?

A. Saline osmotic laxative
B. Bulk-forming laxative
C. Methylcellulose
D. Stool softener

A

A

RATIONALE:
a. Certain saline osmotic laxatives can lead to fluid and electrolyte imbalances and should not be used in older adults and those with kidney or cardiac disease.

176
Q

A nurse caring for a patient who reports frequent constipation learns the patient uses phosphate and sodium citrate enemas several times weekly. What education would the nurse provide?

Electronic Health Record
PMH: Hypertension, chronic kidney disease

A. “Avoid consuming fiber or roughage in the diet.”
B. “Sedentary activities will be helpful.”
C. “These enemas should be avoided with kidney failure.”
D. “Restrict your fluids to 1,000 mL daily.”

A

C

RATIONALE:
c. Hypertonic phosphate and sodium citrate solution enemas are avoided in patients with renal impairment or reduced renal clearance due to their inability to adequately excrete phosphate. This can result in hyperphosphatemia (Dougherty & Lister, 2015, as cited in Mitchell, 2019b, p. 154).

177
Q

A nurse asks a patient for a stool sample to perform the guaiac test. How does the nurse best explain the purpose of this test?

A. “This test replaces the need for screening colonoscopy.”
B. “We are looking for infectious organisms in your stool.”
C. “The screening assesses for blood in your stool.”
D. “This test assesses for antibodies to colon cancer.”

A

C

RATIONALE:
c. The guaiac fecal occult blood testing (FOBT) is used to detect occult blood in the stool. It is used for initial/early screening for disorders such as cancer and for GI bleeding in conditions such as ulcer disease, inflammatory bowel disorders, and intestinal polyps (MedlinePlus, 2020). A positive FOBT result indicates that abnormal bleeding is occurring somewhere in the digestive tract.